Shared Flashcard Set

Details

Pulmonary Pathophysiology
Old Exams
182
Medical
Graduate
09/22/2009

Additional Medical Flashcards

 


 

Cards

Term

Mr. Bard Parker is a 45-year-old man who underwent emergency repair of a ruptured thoracic aortic aneurysm. Since discharge he has noted that he becomes more short of breath doing his daily activities. Physical examination reveals clear lung fields with decreased air entry and respiratory paradox.

 

1-2. What effects will Mr. Parker's condition have on his total lung capacity and his residual volume?

Definition

1. The respiratory paradox indicates diaphragmatic paralysis.

 

2. The total lung capacity will be decreased due to use of the less efficient accessory muscles and the ascent of the diaphragms into the thoracic cavity due to the negative intrathoracic pressure.

 

3. The muscles of expiration are unaffected and residual volume is unchanged.

 

Term

Mr. Bard Parker is a 45-year-old man who underwent emergency repair of a ruptured thoracic aortic aneurysm. Since discharge he has noted that he becomes more short of breath doing his daily activities. Physical examination reveals clear lung fields with decreased air entry and respiratory paradox.

 

3. Mr. Parker notes that he is much more dyspneic when he carries objects. Why?

Definition

He is using his accessory muscles for lifting his arms and they are therefore less efficient at aiding inspiration.

Term

Mrs. Winton-Salem is a 65-year-old woman with severe COPD. Baseline arterial blood gases on room air are pH 7.36, PCO2 60, PO2 60. She is brought into the emergency room by ambulance in acute respiratory distress. Arterial blood gases are pH 7.12, PCO2 90, PO2 120. She undergoes tracheal intubation with institution of mechanical ventilation. Arterial blood gases while receiving mechanical ventilation on 50% supplemental oxygen are pH 7.52, PCO2 40 and PO2 200.

 

4. Is it likely that the patient was receiving supplemental oxygen when she presented to the ER? Why?

 

Definition

She is receiving supplemental oxygen.

 

If she were receiving room air she would have a negative A-a gradient.

Term

Mrs. Winton-Salem is a 65-year-old woman with severe COPD. Baseline arterial blood gases on room air are pH 7.36, PCO2 60, PO2 60. She is brought into the emergency room by ambulance in acute respiratory distress. Arterial blood gases are pH 7.12, PCO2 90, PO2 120. She undergoes tracheal intubation with institution of mechanical ventilation. Arterial blood gases while receiving mechanical ventilation on 50% supplemental oxygen are pH 7.52, PCO2 40 and PO2 200.

 

5. Why was Mrs. Winston-Salem alkalotic while receiving mechanical ventilation despite a normal arterial  PCO2?

Definition

1. She had previously compensated for the respiratory acidosis by retaining bicarbonate.

 

2. Mechanical ventilation acutely lowered the CO2 and the body cannot excrete the bicarbonate as rapidly so she had a metabolic alkalosis.

Term

Mr. Bannister is a 17-year-old high school senior hoping to get an athletic scholarship in track and field. He comes to you for advice on improving his chances.

 

6. Mr. Bannister would like to know what specific workouts could increase his maximum heart rate.

Definition

Maximum heart rate is determined solely by age.

Term

Mr. Bannister is a 17-year-old high school senior hoping to get an athletic scholarship in track and field. He comes to you for advice on improving his chances.

 

7. Mr. Bannister has heard that professional bicycle racers have been using experimental hemoglobin substitutes to increase their performance and wants to know how it would work.

Definition

Hemoglobin substitutes could improve performance by increasing tissue oxygen delivery and now appear to be the drugs of choice in the professional peloton.

Term

Mr. Tsuris is a 23-year-old woman who suffered head trauma in an automobile accident. She is comatose without spontaneous respiratory efforts while receiving mechanical ventilation. she has been stable for the past week but arterial blood gases today show a pH of 7.28, PCO2 55 and PO2 80 while on 50% supplemental oxygen. Further evaluation reveals multiple pulmonary emboli.

 

8. Why is Ms. Tsuris retaining CO2?

Definition

1. The pulmonary emboli have increased blood flow to areas of the lung not affected by the emboli.

 

2. These areas do not get any increase in ventilation as she is paralyzed and cannot increase her respiratory rate.

Term

Mr. Tsuris is a 23-year-old woman who suffered head trauma in an automobile accident. She is comatose without spontaneous respiratory efforts while receiving mechanical ventilation. she has been stable for the past week but arterial blood gases today show a pH of 7.28, PCO2 55 and PO2 80 while on 50% supplemental oxygen. Further evaluation reveals multiple pulmonary emboli.

 

9. Why is her A-a gradient increased?

Definition

Blood leaving the areas which have increased blood flow but no increase in ventilation has lower oxygen tensions and thus the A-a gradient is increased.

Term

Trying to impress his blind date, Mr. Paul Lin brings her a large bouquet of roses. Unfortunately she is highly allergic to roses and develops severe bronchospasm.

 

10-11. Give two reasons why Mr. Lin's date has a marked increase in her minute ventilation following the onset of the bronchospasm.

Definition

1. The increase in ventilation is due to the development of areas of decreased V/Q ratios due to bronchospasm.


2. In order to maintain a normal pCO2 she must increase her minute ventilation.


3. The release of mediators will also increase bronchospasm due to stimulation of irritant receptors.


4. The increased work of breathing will increase CO2 production necessitating an increase in minute ventilation to excrete the excess CO2.

 

Term

You are reading pulmonary function tests as part of your pulmonary elective. Mr. Lannaec has an FVC of 2.3 liters (60% of predicted), and FEV1 of 1.8 liters (58% of predicted) with an FEV1/FVC ratio of 80%. His total lung capacity (TLC) is 3liters (61% predicted). His DICO is 40% of predicted. 

 

12-13. Why are Mr. Lannaec's forced vital capacity and FEV1 decreased?

Definition

1. There is decreased lung compliance decreasing total lung capacity out of proportion to any decrease in residual volume.


2. This decreases the FVC.


3. Although expiratory resistance is normal so airflow during a forced expiratory maneuver is normal, there is less air to exhale so FEV1 is decreased.

Term

You are reading pulmonary function tests as part of your pulmonary elective. Mr. Lannaec has an FVC of 2.3 liters (60% of predicted), and FEV1 of 1.8 liters (58% of predicted) with an FEV1/FVC ratio of 80%. His total lung capacity (TLC) is 3liters (61% predicted). His DICO is 40% of predicted. 

 

14. What is likely to happen to his oxygen saturation with exercise? Why?

Definition

Oxygen saturation will decrease with exercise, as the decreased transit time of the red cell past the alveolus does not allow sufficient time for the hemoglobin to equilibrate with the alveolar gas.

Term

Mr. Eisenmenger is a 65-year-old man with a long-standing ventricular septal defect, which he refuses to have repaired. He now has developed pulmonary hypertension with subsequent right-to-left shunting through the defect.

 

15-16. Give two reasons why Mr. Eisenmenger has a further decrease in his arterial oxygen saturation with exercise.

Definition

1. Increased flow through the VSD

 

and


2. Decreased mixed venous oxygen saturation due to increased oxygen extraction by the exercising tissues.

Term

Mr. Camel is a 70-year-old man with marked dyspnea on exertion. Pulmonary function tests show a forced vital capacity of 1.56 (50% predicted), an FEV1 of 0.75 (34% predicted) and an FEV1/FVC ratio of 48%. His TLC is 5 liters (120% predicted). His DICO is 30% predicted. Arterial blood gases on room air are pH 7.41, PCO2 40, PO2 68.

 

17. Based on these PFT's what is the likely cause of Mr. Camel's airflow obstruction?

Definition

The increased TLC and decreased DlCO indicate emphysema.

Term

Mr. Camel is a 70-year-old man with marked dyspnea on exertion. Pulmonary function tests show a forced vital capacity of 1.56 (50% predicted), an FEV1 of 0.75 (34% predicted) and an FEV1/FVC ratio of 48%. His TLC is 5 liters (120% predicted). His DICO is 30% predicted. Arterial blood gases on room air are pH 7.41, PCO2 40, PO2 68.

 

18. Give two reasons for his decreased DICO.

Definition

1. Decreased surface area for diffusion

 

2. Decreased capillary blood volume.

Term

Mr. Camel is a 70-year-old man with marked dyspnea on exertion. Pulmonary function tests show a forced vital capacity of 1.56 (50% predicted), an FEV1 of 0.75 (34% predicted) and an FEV1/FVC ratio of 48%. His TLC is 5 liters (120% predicted). His DICO is 30% predicted. Arterial blood gases on room air are pH 7.41, PCO2 40, PO2 68.

 

20. Mr. Camel would like to receive supplemental oxygen to reduce his dyspnea. Will this work? Why?

Definition

Mr. Camel's arterial PO2 is above levels that normally stimulate increased ventilation so it is unlikely that it should help his dyspnea.

Term

Mr. Camel is a 70-year-old man with marked dyspnea on exertion. Pulmonary function tests show a forced vital capacity of 1.56 (50% predicted), an FEV1 of 0.75 (34% predicted) and an FEV1/FVC ratio of 48%. His TLC is 5 liters (120% predicted). His DICO is 30% predicted. Arterial blood gases on room air are pH 7.41, PCO2 40, PO2 68.

 

21-23. Give 3 reasons why low, flat diaphragms are inefficient.

Definition

1. Shorter and therefore less able to generate tension


2. Larger radius of curvature and therefore less able to transform tension into pressure


3. Inward movement of the lower rib cage decreasing thoracic volume (Hoover's sign).

Term

Mr. Claus has obstructive sleep apnea, which has severely interfered with his seasonal job making worldwide deliveries. As nasal CPAP machines were unable to function in the extreme environment where he lives he underwent tracheostomy.

 

24. Why did the tracheostomy relieve his symptoms?

Definition

The tracheostomy bypasses the upper airway obstruction that occurs with sleep.

Term

Mr. Benz is a 34-year-old man who develops ARDS following an automobile accident in which he sustains multiple injuries. A pulmonary artery catheter is placed to aid in management. The following measurements are obtained over the next 3 hours. He has a hemoglobin of 10.

 

25. Which level of PEEP is optimal and why?

 

PEEP (cm H2O)

Arterial blood PO2

Arterial oxygen saturation (%)

Cardiac output

0

45

72

4.5

5

50

78

3.8

10

65

90

3.4

15

85

95

2.8

Definition

Although zero PEEP gives the best tissue oxygen delivery many of you were rightly concerned about maintaining such a low PO2 so I accepted 10 as the answer as well.

Term

A little-known Greek myth tells the story of a medical student who angers the God of Pulmonary Medicine. As a punishment the student loses the use of all her inspiratory muscles. Being the typical intelligent Wash U student she is still able to breathe.


1. How can she use her expiratory muscles to generate a tidal volume?

 

Definition

By exhaling to residual volume and then using the outwardly directed recoil properties of the chest wall to "inhale" to FRC.

Term

Mr. Winston is a 67-year-old man, with a 100 pack-year smoking history. He presents with increasing dyspnea that has progressed to the point where he can no longer walk down the street to buy cigarettes. He denies any significant cough or sputum production. On exam he has a barrel shaped chest with a positive Hoover's sign. The point of maximal impulse of the heart is in the subxiphoid space. Arterial blood gases drawn on room air show a pH of 7.40, PCO2 of 40 and PO2 of 64.


2. Mr. Winston notes that he becomes markedly more dyspneic when he is carrying groceries. Why?

 

Definition

1. Mr. Winston has a low flat diaphragm and thus relies on his accessory muscles even during minimal exercise.


2. When these muscles are used to wash his hair, they are less able to aid in inspiration.

Term

Mr. Winston is a 67-year-old man, with a 100 pack-year smoking history. He presents with increasing dyspnea that has progressed to the point where he can no longer walk down the street to buy cigarettes. He denies any significant cough or sputum production. On exam he has a barrel shaped chest with a positive Hoover's sign. The point of maximal impulse of the heart is in the subxiphoid space. Arterial blood gases drawn on room air show a pH of 7.40, PCO2 of 40 and PO2 of 64.


 

3. At night, Mr. Winston's oxygen saturation decreases to 84% from 91%. Why?

 

 

Definition

During sleep the accessory muscles, unlike the diaphragm, lose tone and he hypoventilates.

Term

Mr. Winston is a 67-year-old man, with a 100 pack-year smoking history. He presents with increasing dyspnea that has progressed to the point where he can no longer walk down the street to buy cigarettes. He denies any significant cough or sputum production. On exam he has a barrel shaped chest with a positive Hoover's sign. The point of maximal impulse of the heart is in the subxiphoid space. Arterial blood gases drawn on room air show a pH of 7.40, PCO2 of 40 and PO2 of 64.


4.            Mr. Winston has a decreased DlCO. Why?

 

Definition

Mr. Winston has lost surface area, and capillary blood volume due to his emphysema.

Term

John Manville has asbestosis and consequent severe interstitial lung disease.


5-7.        What three findings on pulmonary function tests will enable you to distinguish Mr. Manville from Mr. Winston?

Definition

Manville

Winston

FEV1/FVC normal or increased

Decreased FEV1/FVC ratio

TLC decreased

TLC increased

FRC decreased

FRC increased

RV decreased or normal

RV increased


Note that both men will have reductions in their FEV1 and FVC it is the ratio that will serve to differentiate their conditions.

Term

Mr. Spuds is a sedentary 45-year-old man whose best friend just suffered a myocardial infarction.  Mr. Spuds wants an exercise test prior to embarking on an exercise program to see if it is safe for him to exercise and determine a baseline level of fitness. He returns after six months of a program of vigorous exercise asking for a repeat test.


8. How will his maximal heart rate have changed? Why?

 

 

Definition
Maximal heart rate depends solely on age and will therefore not significantly change.
Term

Mr. Spuds is a sedentary 45-year-old man whose best friend just suffered a myocardial infarction.  Mr. Spuds wants an exercise test prior to embarking on an exercise program to see if it is safe for him to exercise and determine a baseline level of fitness. He returns after six months of a program of vigorous exercise asking for a repeat test.


 

9.  What effect will the exercise program have on his maximal oxygen consumption? Why?

 

Definition
Maximal oxygen consumption increases due to increased cardiac output and improved oxygen uptake in the tissues.
Term

Mr. Starling is a 70-year-old man intubated and mechanically ventilated after developing pulmonary edema due to an acute myocardial infarction. He required pharmacologic paralysis to facilitate mechanical ventilation. Arterial blood gases yesterday evening showed a pH of 7.41, PCO2 of 38 and PO2 of 120 while receiving 40% oxygen. This morning, on the same yentilator settings the arterial blood gasses showed a pH of 7.60, PCO2 of 28 and PO2 of 160.


10.  Why did Mr. Starling's PCO2 drop?

 

 

Definition
Mr. Starling got better with decreased V/Q mismatch and thus had greater CO2 excretion for the same minute ventilation.
Term

Mr. Starling is a 70-year-old man intubated and mechanically ventilated after developing pulmonary edema due to an acute myocardial infarction. He required pharmacologic paralysis to facilitate mechanical ventilation. Arterial blood gases yesterday evening showed a pH of 7.41, PCO2 of 38 and PO2 of 120 while receiving 40% oxygen. This morning, on the same yentilator settings the arterial blood gasses showed a pH of 7.60, PCO2 of 28 and PO2 of 160.


11.          Why is he alkalotic?

 

Definition
Mr. Starling has both a respiratory and metabolic alkalosis (the change in pH was greater than expected for the change in PCO2)
Term

Michael Plasma is a 35-year-old man who develops a community-acquired pneumonia. Arterial blood gases show a pH of 7.45, PCO2 of 32 and PO2 of 62 while breathing room air. A chest radiograph shows a right lower lobe bronchopneumonia.


12. Why is the PCO2 decreased?

 

Definition

1. The pneumonia stimulated ventilation via the irritant receptors.


2. (As the PO2 was borderline, I also accepted that he hyperventilated to avoid hypoxemia).

Term

Michael Plasma is a 35-year-old man who develops a community-acquired pneumonia. Arterial blood gases show a pH of 7.45, PCO2 of 32 and PO2 of 62 while breathing room air. A chest radiograph shows a right lower lobe bronchopneumonia.


13. Mr. Plasma refuses supplemental oxygen, stating that it will worsen his ventilation perfusion mismatch and cause a respiratory acidosis. Explain why this will not happen.

 

Definition

Although the V/Q mismatch will increase, Mr. Plasma is able to increase his minute ventilation to compensate.

Term

Ms. Hamm is a 20-year-old soccer player who is involved in a collision with another player. She presents to the Emergency Room with chest pain. X-rays reveal several broken ribs on the right. While waiting for her discharge instructions she becomes acutely dyspneic. Vital signs show a heart rate of 130. Arterial blood gases show a pH of 7.42 a PCO2 of 38 and a PO2 of 65 on room air. A repeat chest x-ray shows that there has been accumulation of a large pleural effusion on the right.  Thoracentesis reveals the fluid to be blood.


14.    Why is Ms. Hamm hypoxemic?

 

 

Definition

Ms. Hamm is hypoxemic due to V/Q mismatch caused by compression of the underlying lung.

Term

Ms. Hamm is a 20-year-old soccer player who is involved in a collision with another player. She presents to the Emergency Room with chest pain. X-rays reveal several broken ribs on the right. While waiting for her discharge instructions she becomes acutely dyspneic. Vital signs show a heart rate of 130. Arterial blood gases show a pH of 7.42 a PCO2 of 38 and a PO2 of 65 on room air. A repeat chest x-ray shows that there has been accumulation of a large pleural effusion on the right.  Thoracentesis reveals the fluid to be blood.

 


15. How does the pleural effusion affect the function of her diaphragm?

 

Definition

The diaphragm is flatter and shorter due to the weight of the effusion.

Term

You are a harried intern working up your fifth admission in three hours - a 17-year-old man admitted with diabetic ketoacidosis. After multiple unsuccessful attempts at obtaining an arterial blood gas you decide to follow venous blood gases. Despite several increases in the level of inspired oxygen (FIO2) his venous PO2 remains at 34.


16.  Why does the venous PO2 remain the same?

 

Definition

1. Oxygen delivery remains the same as increases in inspired O2 do not increase the amount of oxygen delivered (the hemoglobin is on the flat part of the oxyhemoglobin dissociation curve).

 

2. As tissue oxygen demand remains constant, the same amount of oxygen is removed and the venous PO2 remains constant.

Term

Ms. Ferrous is a 24-year-old triathlete who has noted a decrease in her endurance. She is very concerned and presents for evaluation. An exercise test confirms her maximal oxygen consumption has decreased significantly from previously measured values. An extensive workup reveals that she has primary pulmonary hypertension.


17. Why does pulmonary hypertension decrease maximal oxygen consumption?

 

Definition

The pulmonary hypertension impedes right heart output to the left ventricle and thus decreases maximum cardiac output.

Term

Mr. Pector is a life-long non-smoker who returns for follow-up after undergoing removal of his right lung due to lung cancer. He is feeling well and has returned to his usual daily activities. Pulmonary function tests reveal an FVC of 2.54 liters (56% predicted) and FEV1of 1.90 liters (54% predicted) and an FEV1/FVC ratio of .75 (100% predicted).


18. Mr. Pector's arterial PO2 is normal. Why?

 

 

Definition

Mr. Pector is able to increase ventilation to the remaining lung to match the increase in blood flow thus maintaining normal V/Q ratios.

Term

Mr. Pector is a life-long non-smoker who returns for follow-up after undergoing removal of his right lung due to lung cancer. He is feeling well and has returned to his usual daily activities. Pulmonary function tests reveal an FVC of 2.54 liters (56% predicted) and FEV1of 1.90 liters (54% predicted) and an FEV1/FVC ratio of .75 (100% predicted).


 

19. Explain the reduction in his FEV1.

 

Definition

1. Mr. Pector has less air to blow out as he has only one lung.


2. The airflow remains normal however

Term

Mr. Pector is a life-long non-smoker who returns for follow-up after undergoing removal of his right lung due to lung cancer. He is feeling well and has returned to his usual daily activities. Pulmonary function tests reveal an FVC of 2.54 liters (56% predicted) and FEV1of 1.90 liters (54% predicted) and an FEV1/FVC ratio of .75 (100% predicted).


20.   What will his DLCO be and why?

 

Definition

It will be decreased reflecting the loss of surface area.

Term

Dr. Lannaec is attending a pulmonary conference in Denver. During a break he visits a booth selling pulse oximeters, which measure arterial oxygen saturation non-invasively. His oxygen saturation is 94%. Back home in St. Louis a similar device had measured his oxygen saturation to be 97%.


21. Explain the difference in readings.

 

Definition

The atmospheric pressure is lower in Denver thus decreasing alveolar PO2.

 

Term

Dr. Lannaec is attending a pulmonary conference in Denver. During a break he visits a booth selling pulse oximeters, which measure arterial oxygen saturation non-invasively. His oxygen saturation is 94%. Back home in St. Louis a similar device had measured his oxygen saturation to be 97%.


22. Would you expect the arterial PCO2 to be different between St. Louis and Denver? Why?

Definition

1. It will remain the same as none of the determinants of arterial PCO2 are altered.


2. (I also accepted that it will decrease due to hyperventilation to maintain a normal PO2 although the decrease would likely not be enough to stimulate hyperventilation.)

Term

Mr. Mortis is a 70-year-old man who dislikes physicians and thus has received no medical care for over 50 years. His family brings him to the ER stating that he developed a cold two days ago. Today they found him in bed, confused and sleepy. Arterial blood gases showed a pH of 7.20, PCO2 of 95 and PO2 of 100. Mr. Mortis is intubated and placed on mechanical ventilation.


23.  Is Mr. Mortis likely to have chronically retained CO2? Why?

 

 

Definition

1. Chronic.


2. His pH is higher than expected if the change in PCO2 were acute.

Term

Mr. Mortis is a 70-year-old man who dislikes physicians and thus has received no medical care for over 50 years. His family brings him to the ER stating that he developed a cold two days ago. Today they found him in bed, confused and sleepy. Arterial blood gases showed a pH of 7.20, PCO2 of 95 and PO2 of 100. Mr. Mortis is intubated and placed on mechanical ventilation.


24. Was Mr. Mortis breathing room air when these blood gases were drawn? Why?

 

Definition

1. No.


2. The A-a gradient would be negative if he were breathing room air.

Term

Mr. Mortis is a 70-year-old man who dislikes physicians and thus has received no medical care for over 50 years. His family brings him to the ER stating that he developed a cold two days ago. Today they found him in bed, confused and sleepy. Arterial blood gases showed a pH of 7.20, PCO2 of 95 and PO2 of 100. Mr. Mortis is intubated and placed on mechanical ventilation.

 

25.   What will happen to his pH if mechanical ventilation decreases his arterial PCO2 to 40? Why?

Definition

He would be alkalotic as he is chronically retaining bicarbonate.

Term

Ms. Wolf is a 35-year-old woman with a history of increasing dyspnea on exertion associated with a dry cough, night sweats, joint pain, and recent onset of pleuritic chest pain. Pulmonary function tests reveal a forced vital capacity (FVC) of 3.0 liters (60%-predicted) and a forced expiratory volume in one second (FEV1) of 2.5 liters (62% predicted).


1. On further testing her total lung capacity is reduced. What physiologic change could cause this?

 

 

 


 

Definition

She has decreased lung compliance making it more difficult for the inspiratory muscles to expand the lungs and chest wall.

Term

Ms. Wolf is a 35-year-old woman with a history of increasing dyspnea on exertion associated with a dry cough, night sweats, joint pain, and recent onset of pleuritic chest pain. Pulmonary function tests reveal a forced vital capacity (FVC) of 3.0 liters (60%-predicted) and a forced expiratory volume in one second (FEV1) of 2.5 liters (62% predicted).

 

2.  Predict whether her residual volume will be increased or decreased? Why?

Definition

1. Her residual volume will be decreased. 


2. The increased tethering effect of the parenchyma on the small airways will keep them open to lower lung volumes.

Term

Ms. Wolf is a 35-year-old woman with a history of increasing dyspnea on exertion associated with a dry cough, night sweats, joint pain, and recent onset of pleuritic chest pain. Pulmonary function tests reveal a forced vital capacity (FVC) of 3.0 liters (60%-predicted) and a forced expiratory volume in one second (FEV1) of 2.5 liters (62% predicted).

 

3. What is the physiology (not disease) of her decrease in forced vital capacity?

Definition

Her forced vital capacity will decrease because the total lung capacity is decreased to a greater percentage than the residual volume.

Term

Mr. Willebrand is a 65-year-old man with emphysema who is recuperating from a broken leg. He suddenly becomes dyspneic, tachycardic and less responsive. His blood pressure is 80/50, pulse is 120, and respiratory rate is 30. Arterial blood gases reveal a pH of 7.45, PCO2 of 32 and PO2 of 45 on 2 liters of supplemental oxygen. Previous blood gases were pH of 7.40, PCO2 of 40 and PO2 of 60 on room air. A pulmonary angiogram showed a pulmonary embolus occluding the artery to the right posterior basal segment. With therapy his blood pressure improves to 130/70. Arterial blood gases drawn while receiving an inspired oxygen concentration (FIO2) of 50% reveal a pH of 7.46, PCO2 of 32 and PO2 of 160.

 

4. Despite a small embolus, Mr. Willebrand is hypotensive. Why may this happen?         

5.  Mr. Willebrand continues to be alkalotic despite correction of his hypoxemia. Why?

Definition

1. Mr. Willebrand’s preexisting lung disease has decreased the number of capillaries available for him to recruit

 

2. The remaining capillaries may not be able to dilate further in response to the increase in flow that they receive. 

 

3. Pulmonary vascular resistance will therefore not decrease

 

4. Pulmonary artery pressure will rise, interfering with (a) right ventricular output and (b) left ventricular filling.

Term

Mr. Willebrand is a 65-year-old man with emphysema who is recuperating from a broken leg. He suddenly becomes dyspneic, tachycardic and less responsive. His blood pressure is 80/50, pulse is 120, and respiratory rate is 30. Arterial blood gases reveal a pH of 7.45, PCO2 of 32 and PO2 of 45 on 2 liters of supplemental oxygen. Previous blood gases were pH of 7.40, PCO2 of 40 and PO2 of 60 on room air. A pulmonary angiogram showed a pulmonary embolus occluding the artery to the right posterior basal segment. With therapy his blood pressure improves to 130/70. Arterial blood gases drawn while receiving an inspired oxygen concentration (FIO2) of 50% reveal a pH of 7.46, PCO2 of 32 and PO2 of 160.

       

5.  Mr. Willebrand continues to be alkalotic despite correction of his hypoxemia. Why?

Definition

1. Stimulation of the irritant and juxtacapillary receptors in the lung. 


2. Potentially, the pulmonary hypertension may also stimulate hyperventilation.

Term

Mr. Nocturne is a 50-year-old truck driver who has had three accidents in the past year. Physical examination reveals a weight of 350 pounds, and a blood pressure of 170/102 but is otherwise normal. A sleep study confirms severe sleep apnea.


6. Describe the sequence of events that leads to upper airway obstruction in sleep apnea.

 

 

Definition

1. There is loss of tone in the muscles of the upper airway.

 

2. This causes the resultant occlusion of the airway by the tongue and soft palate.

 

3. Increasing diaphragmatic contractions worsen the obstruction.

Term

Mr. Nocturne is a 50-year-old truck driver who has had three accidents in the past year. Physical examination reveals a weight of 350 pounds, and a blood pressure of 170/102 but is otherwise normal. A sleep study confirms severe sleep apnea.

 

7.  In the absence of therapy, what relieves the obstruction?

Definition

Mr. Nocturne awakens and tone returns to the upper airway muscles.

Term

Mr. Descartes is a 40-year-old man who suddenly loses consciousness after complaining of "the worst headache of my life". On exam his vital signs are a blood pressure of 180/100 with a pulse of 50 and respiratory rate of 6. He is unresponsive to deep pain, and has unequal pupils that are sluggishly responsive to light. Fundoscopy reveals swelling of the optic disk. Arterial blood gases are pH of 7.16, PCO2 of 70 and PO2 of 60 while breathing room air.


8.  Is it likely that Mr. Descartes chronically retains CO2? Why?


 

Definition

1. It is unlikely that Mr. Descartes is a chronic CO2 retainer 

 

2. The pH change is proportional to the change in PCO2.

Term

Mr. Descartes is a 40-year-old man who suddenly loses consciousness after complaining of "the worst headache of my life". On exam his vital signs are a blood pressure of 180/100 with a pulse of 50 and respiratory rate of 6. He is unresponsive to deep pain, and has unequal pupils that are sluggishly responsive to light. Fundoscopy reveals swelling of the optic disk. Arterial blood gases are pH of 7.16, PCO2 of 70 and PO2 of 60 while breathing room air.


9. Why does he have a low arterial PO2?

Definition

Hypoventilation (the A-a gradient is normal).

Term

Ms. Terry is a 25-year-old woman who has the following values on her pulmonary function test:


 

Test

Value

% predicted normal

Forced vital capacity (FVC)

5 liters

100%

Forced expiratory volume in one second (FEV1)

4 liters

100%

FEV1/FVC

80%

100%

Total lung capacity (TLC)

6.6 liters

100%

Diffusing capacity (DlCO)

20 ml/min/mm Hg

65%

 

 

10-11.    Give two possible explanations for the findings.

 

Definition

Mrs. Terry may be:

 

1. Anemic

 

2. Have suffered pulmonary emboli

 

3. Have a high level of carbon monoxide in her blood

 

Term

Mr. Mortis is receiving mechanical ventilation following surgery for a traumatic rupture of his aorta. He developed delirium tremens and is now heavily sedated and is receiving paralytic agents. The high-pressure alarm on his ventilator suddenly sounds indicating that he requires a higher pressure to receive the preset tidal volume and his oxygen saturation drops to 80% from the previous value of 95%. Arterial blood gases show a pH of 7.30. PCO2 of 50 and PO2 of 50 on an FIO2 of 40%. Exam shows no breath sounds on the left. Chest radiographs show that his endotracheal tube is now in his right mainstem bronchus.


12. Why did he require a higher airway pressure to get the same tidal volume?

Definition

1. The tidal volume that was being delivered to both lungs is now delivered to just one. 

 

2. Therefore, the alveoli are receiving a: -Greater change in volume and require a -Greater change in pressure.

Term

Mr. Mortis is receiving mechanical ventilation following surgery for a traumatic rupture of his aorta. He developed delirium tremens and is now heavily sedated and is receiving paralytic agents. The high-pressure alarm on his ventilator suddenly sounds indicating that he requires a higher pressure to receive the preset tidal volume and his oxygen saturation drops to 80% from the previous value of 95%. Arterial blood gases show a pH of 7.30. PCO2 of 50 and PO2 of 50 on an FIO2 of 40%. Exam shows no breath sounds on the left. Chest radiographs show that his endotracheal tube is now in his right mainstem bronchus.

 


13. Why did his arterial PO2 drop?

Definition

The blood going to the left lung was not receiving any ventilation making it a shunt

Term

Mr. Mortis is receiving mechanical ventilation following surgery for a traumatic rupture of his aorta. He developed delirium tremens and is now heavily sedated and is receiving paralytic agents. The high-pressure alarm on his ventilator suddenly sounds indicating that he requires a higher pressure to receive the preset tidal volume and his oxygen saturation drops to 80% from the previous value of 95%. Arterial blood gases show a pH of 7.30. PCO2 of 50 and PO2 of 50 on an FIO2 of 40%. Exam shows no breath sounds on the left. Chest radiographs show that his endotracheal tube is now in his right mainstem bronchus.

 


14. What effect would increasing the FIO2 have on his arterial PO2? Why?

Definition

There should be little effect as the hypoxemia is due to shunt

Term

Mr. Mac Heath has developed a pneumothorax following placement of a subclavian catheter. He is asymptomatic. Arterial blood gases. which were previously normal are now pH of 7.42, PCO2 of 36 and PO2 of 68.


15.  Why did his arterial PO2 decrease?

 

Definition

The lungs have compression of alveoli, which cause ventilation and perfusion mismatch.

Term

Mr. Mac Heath has developed a pneumothorax following placement of a subclavian catheter. He is asymptomatic. Arterial blood gases. which were previously normal are now pH of 7.42, PCO2 of 36 and PO2 of 68.

 


16. How will supplemental oxygen speed the resolution of the pneumothorax?

Definition

1. The decrease in nitrogen in the arterial and capillary blood increases the pressure gradient

 

2. This favors reabsorption of the air from the pleural space.

Term

Mr. Taussig is a 21-year-old man who presents because he noted that he looked blue after exercising. A careful exam reveals a prominent right ventricular impulse, an increase splitting of the second heart sound with a loud pulmonic component and a 216 ejection murmur at the upper left sternal border. An echocardiogram confirms an atrial septal defect with pulmonary hypertension and right to left shunt.


17-18.    Give two reasons why Mr. Taussig's oxygen saturation decreases with exercise.

 

Definition

1. There will be a decrease in the mixed venous oxygen during exercise. 


2. As he has a right-to-left shunt, the arterial PO2 is greatly affected by the mixed venous oxygen tension. 


3. Additionally, the further increase in pulmonary artery pressures that occur in this patient with exercise will increase the shunt flow through the ASD.

Term

Ms. Dander is a 16-year-old woman with asthma. She has recently developed increasing cough and sputum production and has had to increase her use of inhaled beta-agonist bronchodilators. She now presents to the emergency room severely dyspneic. On exam she is using all accessory muscles to breath and is somnolent. Her respiratory rate is 40. She has no wheezes and has no pulsus paradox. Her arterial blood gases while breathing 100% oxygen are pH of 7.10, PCO2 of 75 and PO2 of 200. She is intubated and placed on mechanical ventilation. Despite further administration of beta-agonist bronchodilators she requires high airway pressures to deliver a tidal volume of 500cc. Examination now reveals diffuse wheezes.


19.          Ms. Dander has an elevated PCO2 despite a very high respiratory rate. Why?

 

Definition

1. Ms. Dander has severe ventilation perfusion mismatch that necessitates a high minute ventilation in order to try and maintain a normal PCO2.

 

2. She has a high work of breathing increasing oxygen consumption and, hence, CO2 production. 

 

3. She is breathing rapidly and shallowly, which increases the VD/VT ratio making her ventilation less efficient. 

 

4. Thus despite her tachypnea the alveolar ventilation is insufficient to be able to maintain her CO2 in the normal range. 

 

5. It is this imbalance between CO2 production and alveolar ventilation which increases the CO2.

Term

Ms. Dander is a 16-year-old woman with asthma. She has recently developed increasing cough and sputum production and has had to increase her use of inhaled beta-agonist bronchodilators. She now presents to the emergency room severely dyspneic. On exam she is using all accessory muscles to breath and is somnolent. Her respiratory rate is 40. She has no wheezes and has no pulsus paradox. Her arterial blood gases while breathing 100% oxygen are pH of 7.10, PCO2 of 75 and PO2 of 200. She is intubated and placed on mechanical ventilation. Despite further administration of beta-agonist bronchodilators she requires high airway pressures to deliver a tidal volume of 500cc. Examination now reveals diffuse wheezes.


 

20.          Why is Ms. Dander's airflow obstruction not responding to bronchodilator therapy?

 

Definition

The airway obstruction is primarily due to:

 

1. Mucosal edema and

 

2. Inspissated secretions

 

rather than bronchospasm.

Term

Ms. Dander is a 16-year-old woman with asthma. She has recently developed increasing cough and sputum production and has had to increase her use of inhaled beta-agonist bronchodilators. She now presents to the emergency room severely dyspneic. On exam she is using all accessory muscles to breath and is somnolent. Her respiratory rate is 40. She has no wheezes and has no pulsus paradox. Her arterial blood gases while breathing 100% oxygen are pH of 7.10, PCO2 of 75 and PO2 of 200. She is intubated and placed on mechanical ventilation. Despite further administration of beta-agonist bronchodilators she requires high airway pressures to deliver a tidal volume of 500cc. Examination now reveals diffuse wheezes.


21.          Why was her pulsus paradox not increased on presentation?

 

Definition

Due to diaphragmatic fatigue she was unable to generate sufficient intrathoracic pressure to cause an increase in venous return to the right heart.

Term

Mr. Indurain is a 60-year old well-trained endurance athlete. Mr. Spud is a 30-year-old healthy but sedentary individual.


22. Who will have the higher maximum heart rate? Why?

 

 

Definition

Mr. Spud as maximum heart rate is determined by age.

Term

Mr. Indurain is a 60-year old well-trained endurance athlete. Mr. Spud is a 30-year-old healthy but sedentary individual.



23. Who will have the higher maximum oxygen consumption? Why?

Definition

1. Mr. Indurain will have the higher maximum oxygen consumption

 

He will be better able to

1. Maximize stroke volume and

2. Extract oxygen at the tissue level.

Term

Mr. Indurain is a 60-year old well-trained endurance athlete. Mr. Spud is a 30-year-old healthy but sedentary individual.


24. Who will have the lower mixed venous oxygen level at maximum exercise? Why?

Definition

Mr. Indurain has:

1. Increased muscle capillaries

2. Altered oxidative enzymes and a

3. Higher cardiac output allowing a greater percentage of his cardiac output to go to the exercising muscles. 

 

These will allow him to:

1. Maximally extract oxygen and

2. Give him a lower mixed venous oxygen tension at maximum exercise.

Term

Mr. Indurain is a 60-year old well-trained endurance athlete. Mr. Spud is a 30-year-old healthy but sedentary individual.

 

25.  How will using supplemental oxygen during exercise affect Mr. Spuds maximum oxygen consumption?

Definition

1. Supplemental oxygen will not significantly increase tissue oxygen delivery 

 

2. He is on the flat portion of his oxyhemoglobin dissociation curve and

 

2. Will therefore not change his maximum oxygen consumption.

Term

You are working part-time in a fitness store when a customer comes in wanting to return a treadmill she has been using regularly for two months. She states that it is no longer working correctly because he can no longer reach her target heart rate on the same settings she has been using since she bought it.


1.            Explain to the customer what has happened.

 

Definition

1. The customer has become more fit and therefore can accomplish the same amount of energy expenditure with a lower heart rate.

 

2. Her exercise regimen has enabled her to increase her stroke volume and peripheral oxygen extraction. She can thus achieve the same oxygen consumption at a lower heart rate.

Term

You are on morning ICU rounds when the nurse hands you a set of arterial blood gases drawn on Mr. Sweet, a 25-year-old man admitted 24 hours ago with diabetic ketoacidosis. He was treated with intravenous fluids and insulin. They are pH 7.47, PCO2 30 and PO2 80 on room air. He states that he is feeling well and is not short of breath.


2.   Why is does he have a respiratory alkalosis?

 

Definition

1. Mr. Sweet's metabolic acidosis has resolved but there has been insufficient time for the bicarbonate to equilibrate between the blood and the CNS across the blood-brain barrier.

 

2. The increase in peripheral pH causes a decrease in the minute ventilation through the carotid receptors.

 

3. The resulting increase in PCO2 does equilibrate across the blood-brain barrier decreasing the CNS pH, therefore increasing minute ventilation.

 

1. He has a paradoxical CSF acidosis due to the slow diffusion of bicarbonate across the blood-brain barrier.

 

2. This continues to stimulate his central chemoreceptors.

Term

Mr. Camel is a 55-year-old smoker with progressive dyspnea on exertion and swelling of his legs. Physical examination shows distension of the jugular veins to the angle of the jaw, diffuse wheezes throughout all lung fields, a prominent pulmonic component to S2, a liver edge palpable 3 cm below the right costal margin, and pitting edema of the legs extending to the knees. Arterial blood gases on room air show pH 7.36, PCO2 65, PO2 40. He is given supplemental oxygen and repeat arterial blood gases on an FIO2 of 30% show pH 7.32, PCO2 70 and PO2 56.


3.  Is his CO2 retention acute or chronic? Why?

Definition

1. Mr. Camel has chronic CO2 retention as the decrease in his pH is less than predicted for the rise in his PCO2.


1. Chronic, as the pH is not decreased in the expected proportion to the increased PCO2.

Term

Mr. Camel is a 55-year-old smoker with progressive dyspnea on exertion and swelling of his legs. Physical examination shows distension of the jugular veins to the angle of the jaw, diffuse wheezes throughout all lung fields, a prominent pulmonic component to S2, a liver edge palpable 3 cm below the right costal margin, and pitting edema of the legs extending to the knees. Arterial blood gases on room air show pH 7.36, PCO2 65, PO2 40. He is given supplemental oxygen and repeat arterial blood gases on an FIO2 of 30% show pH 7.32, PCO2 70 and PO2 56.

 

4. Why does he have pulmonary hypertension?

 

Definition

1. He has hypoxemia and hypercapnia. It is unlikely from his clinical presentation that he has any emphysema.


1. Hypoxemia and hypercarbia.

Term

Mr. Camel is a 55-year-old smoker with progressive dyspnea on exertion and swelling of his legs. Physical examination shows distension of the jugular veins to the angle of the jaw, diffuse wheezes throughout all lung fields, a prominent pulmonic component to S2, a liver edge palpable 3 cm below the right costal margin, and pitting edema of the legs extending to the knees. Arterial blood gases on room air show pH 7.36, PCO2 65, PO2 40. He is given supplemental oxygen and repeat arterial blood gases on an FIO2 of 30% show pH 7.32, PCO2 70 and PO2 56.


 

5-6. Give two reasons why his PCO2 increased when he was given supplemental oxygen.

Definition

1. The supplemental oxygen released hypoxic pulmonary vasoconstriction increasing perfusion to alveoli, which did not increase ventilation.


2. He did not increase his minute ventilation to compensate for this increase in V/Q mismatch.


3. Likewise, he did not increase his minute ventilation to compensate for the extra CO2 released from better oxygenated hemoglobin due to the Haldane effect.



1. Increased V/Q mismatch due to release of hypoxic vasoconstriction without an increase in minute ventilation.


2. The Haldane effect, again without an increase in minute ventilation

Term

You are reading pulmonary function tests as part of your pulmonary elective. The following are the values for Mr. Pith who is 24 years old.


 

 

Value

% predicted normal

Forced vital capacity (FVC)

3.33 liters

66%

Forced expiratory volume in one second (FEV1)

1.5 liters

37%

FEV1/FVC

45%

56%

Functional residual capacity (FRC)

2.5 liters

97%

Residual volume (RV)

2.5 liters

285%

Total lung capacity (TLC)

7.0 liters

93%

 

 

7. What is the most likely etiology of Mr. Pith's abnormal pulmonary function tests?

Definition

1. The elevation of the RV to the value of the FRC indicates inability to exhale actively.


2. His TLC is relative normal indicating sparing of the inspiratory muscles.


3. This combination is seen in patients with spinal cord lesions below C6.



1. Transection of the spinal cord at the C6 level.


2. The RV and FRC are exactly the same indicating that he has lost the use of his expiratory (not accessory) muscles.

Term

You are reading pulmonary function tests as part of your pulmonary elective. The following are the values for Mr. Pith who is 24 years old.


 

 

Value

% predicted normal

Forced vital capacity (FVC)

3.33 liters

66%

Forced expiratory volume in one second (FEV1)

1.5 liters

37%

FEV1/FVC

45%

56%

Functional residual capacity (FRC)

2.5 liters

97%

Residual volume (RV)

2.5 liters

285%

Total lung capacity (TLC)

7.0 liters

93%

 

 

 

8.   Why is the FEV1/FVC ratio reduced?

Definition

1. He has difficulty giving a good expiratory effort due to paralysis of his expiratory muscles.


 


1. He is unable to forcefully exhale the air from his lungs so airflow is markedly reduced.

 

Term

You are reading pulmonary function tests as part of your pulmonary elective. The following are the values for Mr. Pith who is 24 years old.


 

 

Value

% predicted normal

Forced vital capacity (FVC)

3.33 liters

66%

Forced expiratory volume in one second (FEV1)

1.5 liters

37%

FEV1/FVC

45%

56%

Functional residual capacity (FRC)

2.5 liters

97%

Residual volume (RV)

2.5 liters

285%

Total lung capacity (TLC)

7.0 liters

93%

 

 

9.   Is he likely to exhibit Hoover's sign on physical exam? Why?

Definition

1. No, as a Hoover's sign indicates non-functioning diaphragms.

2. No, as he is not hyperinflated.

Term

Ms. Perry Natal is a 32-year-old woman who developed ARDS following delivery of her baby. She was chemically paralyzed to facilitate mechanical ventilation. She is febrile to 103.6 degrees. Her hemoglobin is decreased at 8.0 g/dL (normal range is 13.5 - 15 g/dL), her cardiac output is 12 L/minute (normal values at rest are 4 - 6 L/minute). Arterial blood gases obtained on an FIO2 of 100% are pH 7.43, PCO2 36 and PO2 70 with an oxygen saturation of 96%. Her mixed venous PO2 is 28 with an oxygen saturation of 62%.


10-11. Give two interventions that could increase the mixed venous PO2 in this patient.

Definition

1. Decrease oxygen consumption by decreasing her fever, increase the hemoglobin to improve delivery.


2. Her cardiac output is already very high and her hemoglobin is virtually maximally saturated.



1. Increase her hemoglobin.


2. Decrease her fever.


3. PEEP is not likely to increase her mixed venous oxygen as she is already on the flat part of the oxyhemoglobin dissociation curve.


4. Any further increase in PO2 will have little effect on SaO2 and thus little effect on tissue oxygen delivery.


5. Indeed, PEEP may decrease cardiac output and therefore decrease mixed venous O2.

Term

Ms. Perry Natal is a 32-year-old woman who developed ARDS following delivery of her baby. She was chemically paralyzed to facilitate mechanical ventilation. She is febrile to 103.6 degrees. Her hemoglobin is decreased at 8.0 g/dL (normal range is 13.5 - 15 g/dL), her cardiac output is 12 L/minute (normal values at rest are 4 - 6 L/minute). Arterial blood gases obtained on an FIO2 of 100% are pH 7.43, PCO2 36 and PO2 70 with an oxygen saturation of 96%. Her mixed venous PO2 is 28 with an oxygen saturation of 62%.

 

12. How would the increase in mixed venous PO2 allow you to decrease the inspired oxygen level (FIO2)?

Definition

1. In the presence of shunt, the arterial oxygen tension is dependent on the mixed venous oxygen saturation.



1. The O2 saturation of the blood leaving the shunt units will increase and therefore have less effect on the overall O2 saturation of the mixture of blood from the ventilated and non-ventilated units.


2. The arterial PO2 will therefore increase.

Term

Two days later, on the same ventilator settings as previously, Ms. Natal has arterial blood gases of pH 7.52, PCO2 25, PO2 450.


 13.  Despite the same minute ventilation the PCO2 is lower. Why?

 

Definition

1. Improved V/Q matching as she gets better, allowing the same ventilation to excrete more CO2.

2. She has gotten better with decreased shunt.

Term

Mr. Marlboro is scheduled to have his left lower lobe removed due to lung cancer. He is a life-long non-smoker who has normal pulmonary function tests.


14.          What will happen to his pulmonary artery pressure following surgery? Why?

 

 

Definition

1. He should have sufficient reserve to be able to recruit and dilate the capillaries in his remaining lung to reduce resistance sufficiently to avoid hypertension.

2. It will remain the same as he will lose only approximately 20% of his capillary volume and will therefore have sufficient reserve.

Term

Mr. Marlboro is scheduled to have his left lower lobe removed due to lung cancer. He is a life-long non-smoker who has normal pulmonary function tests.


 

15. What will happen to his DlCO following surgery? Why?

Definition

1. It will decrease as he is losing surface area for diffusion.

1. It will decrease to loss of capillary volume and area for diffusion

Term

Mr. Marlboro is scheduled to have his left lower lobe removed due to lung cancer. He is a life-long non-smoker who has normal pulmonary function tests.


16.          What will happen to the FEV1/FVC ratio following surgery? Why?

Definition

1. It should remain normal as airway resistance should not be affected.


1. It will remain the same as he will have less volume within his remaining lungs but will continue to have normal airflow so the air will be exhaled at the normal flow rate. 

Term

Ms. Mast presents with sudden onset of wheezing and marked shortness of breath following exposure to cat dander. She is using all accessory muscles of inspiration, has a pulsus paradox of 30 mmHg and diffuse wheezing. Chest radiographs show hyperinflated lungs without evidence of pneumonia. Arterial blood gases show pH 7.52, PCO2 of 28 and PO2 of 78 on 4 liters/minute supplemental oxygen delivered by nasal cannula.


17.  Why does Ms. Mast have alveolar hyperventilation?

 

Definition

1. Release of mediators.

1. Release of mediators which are stimulating irritant receptors within the lungs. 

Term

Ms. Mast presents with sudden onset of wheezing and marked shortness of breath following exposure to cat dander. She is using all accessory muscles of inspiration, has a pulsus paradox of 30 mmHg and diffuse wheezing. Chest radiographs show hyperinflated lungs without evidence of pneumonia. Arterial blood gases show pH 7.52, PCO2 of 28 and PO2 of 78 on 4 liters/minute supplemental oxygen delivered by nasal cannula.


 

18.    Why are her lungs hyperinflated?

 

Definition

1. She has insufficient time to exhale all the air before she inhales again.

 

 

2. She has air trapping behind narrow airways and is also unable to exhale all the air before she needs to inhale again. 

Term

Ms. Mast presents with sudden onset of wheezing and marked shortness of breath following exposure to cat dander. She is using all accessory muscles of inspiration, has a pulsus paradox of 30 mmHg and diffuse wheezing. Chest radiographs show hyperinflated lungs without evidence of pneumonia. Arterial blood gases show pH 7.52, PCO2 of 28 and PO2 of 78 on 4 liters/minute supplemental oxygen delivered by nasal cannula.


 

19.    How does the hyperinflation affect the work of breathing?

Definition

1. It increases the work as the inflated lungs are on a less compliant portion of their compliance curve.

 

2. The increased radial traction on the small airways may slightly decrease airway resistance.


1. The lungs are on the less compliant portion of their pressure-volume curve and the   low, flat diaphragm is at a mechanical disadvantage. 

Term

Mr. Indurain is a 25-year-old man who presents following a bicycling accident. He is hypotensive and cyanotic. His exam shows jugular venous distension, shift of the trachea to the right and absent breath sounds on the left with hyperresonance on percussion. A chest radiograph confirms the presence of a left tension pneumothorax.


20.    Why is he hypotensive?

 

Definition

1. The shift of the mediastinum kinks the major vessels impeding blood return to and from the heart.

1. The great vessels are "kinked" thus markedly reducing venous return to the right side   of the heart. 

Term

Mr. Mansville is a 75-year-old man with asbestosis. His pulmonary function tests are compatible with severe restrictive disease. His arterial PO2 at rest is 60 and it decreases during exercise.


21.    Why does he have an elevated A-a gradient at rest?

Definition

1. V/Q mismatch caused by the heterogeneous distribution of the disease.


2. Thus, some areas of the lung have less change in volume and thus less ventilation than other areas for a given change in pleural pressure.


1. Ventilation perfusion mismatch. 

Term

Mr. Mansville is a 75-year-old man with asbestosis. His pulmonary function tests are compatible with severe restrictive disease. His arterial PO2 at rest is 60 and it decreases during exercise.

 

22.  He is given supplemental oxygen, which increases his resting arterial PO2 to 70. Will his arterial PO2 continue to decrease with exercise? Why?

Definition

1. The PO2 will continue to decrease as there will still be insufficient time for the oxygen to equilibrate across the alveolo-capillary membrane.

1. It will continue to decrease, as there will still be insufficient time for equilibration   across the alveolo-capillary membrane. 

Term

Mr. Harvey has increasing dyspnea on exertion. Examination of the lungs reveals bilateral crackles with dullness to percussion and decreased fremitus over the right posterior lung field. Cardiac examination is remarkable for an S3 gallop and diffuse PMI. Chest radiographs confirm congestive heart failure and a large right pleural effusion.


23.          What causes pleural fluid to accumulate in congestive heart failure?

 

Definition

1. Increased hydrostatic pressure in the pleural capillaries.

1. Increased hydrostatic pressure within the pleural capillaries. 

Term

Mr. Harvey has increasing dyspnea on exertion. Examination of the lungs reveals bilateral crackles with dullness to percussion and decreased fremitus over the right posterior lung field. Cardiac examination is remarkable for an S3 gallop and diffuse PMI. Chest radiographs confirm congestive heart failure and a large right pleural effusion.

 

 

24-25.    List two physiologic consequences of pleural effusions.

Definition

1. Increased V/Q mismatch due to compression of the adjacent lung tissue and


2. Decreased diaphragmatic pressure generation as it is flattened and shortened by the overlying fluid.


1. Increased V/Q mismatch due to compression of the lung. 


2. Shortening and flattening of the diaphragm due to the weight of the effusion. 

Term

Mr. Claus is an elderly obese man whose wife states that he has been increasingly sleepy over the past several years. She states that recently he has slept through several important delivery dates. She complains that he snores during the night, and at times appears to stop breathing. Physical examination reveals a jolly, obese man with signs of pulmonary hypertension and cor pulmonale. A sleep study reveals numerous apneic periods during sleep.


1. What is the likely cause of airflow obstruction when he sleeps?

Definition

1. The airflow obstruction is secondary to occlusion of the upper airway by the tongue falling against the posterior pharynx.

Term

Mr. Claus is an elderly obese man whose wife states that he has been increasingly sleepy over the past several years. She states that recently he has slept through several important delivery dates. She complains that he snores during the night, and at times appears to stop breathing. Physical examination reveals a jolly, obese man with signs of pulmonary hypertension and cor pulmonale. A sleep study reveals numerous apneic periods during sleep.


 

2.  What is the likely cause of his pulmonary hypertension?

Definition

Recurrent nocturnal hypoxemia causes his pulmonary hypertension.

Term

Mr. Claus is an elderly obese man whose wife states that he has been increasingly sleepy over the past several years. She states that recently he has slept through several important delivery dates. She complains that he snores during the night, and at times appears to stop breathing. Physical examination reveals a jolly, obese man with signs of pulmonary hypertension and cor pulmonale. A sleep study reveals numerous apneic periods during sleep.


 

3.   Why is he sleepy?

Definition

1. He is sleepy because hypoxemia causes numerous awakenings during the night.


2. These awakenings restart his breathing but rob him of sleep.


3. It is not just REM sleep that is affected, all stages of sleep are disrupted.

Term

Mr. Croupin is a 45-year-old lawyer who presents with an acute onset of chest pain following a vigorous sneeze. Physical examination reveals decreased breath sounds on the right. Chest x-ray reveals a right pneumothorax with partial collapse of the lung. Arterial blood gases showed a pH of 7.40, PCO2 of 40 and PO2 of 64 on room air.


4.  Why may a lung only partially collapse?

Definition

1. As the lung collapses the pressure in the pleural space decreases (Boyle's Law).


2. If the leak is small the negative pressure in the pleural space can counterbalance the recoil pressure of the lung and keep it from totally collapsing.

Term

Mr. Croupin is a 45-year-old lawyer who presents with an acute onset of chest pain following a vigorous sneeze. Physical examination reveals decreased breath sounds on the right. Chest x-ray reveals a right pneumothorax with partial collapse of the lung. Arterial blood gases showed a pH of 7.40, PCO2 of 40 and PO2 of 64 on room air.


 

5. Why is his A-a gradient increased?

 

Definition

The atelectasis in the collapsed lung causes:


1. V/Q mismatch

 

and/or


2. Shunting.

Term

Mr. Croupin is a 45-year-old lawyer who presents with an acute onset of chest pain following a vigorous sneeze. Physical examination reveals decreased breath sounds on the right. Chest x-ray reveals a right pneumothorax with partial collapse of the lung. Arterial blood gases showed a pH of 7.40, PCO2 of 40 and PO2 of 64 on room air.

 

6. How does increasing his FIO2 speed resolution of the pneumothorax?

Definition

1. The higher FIO2 washes out nitrogen from the alveoli and the arterial blood.


2. Although this increases the arterial PO2, it does not significantly increase tissue oxygen delivery.


3. Thus, the capillary PO2 is not significantly increased over its usual level.


4. However, since there is no nitrogen in the capillary blood, there is a marked increase in the gradient favoring absorption of the pleural gases.

Term

Ms. Take is a 24-year-old woman in the midst of an asthmatic attack. She has received multiple doses of bronchodilators but remains severely dyspneic. Arterial blood gases show a pH of 7.42, PCO2 of 30 and PO2 of 54 on 3 liters of supplemental oxygen.


7.  Why is she refractory to bronchodilator therapy?

 

Definition

She is refractory to bronchodilators because the obstruction is primarily due to:


1. Mucosal edema

2. Inflammation and

3. Thick secretions


rather than bronchospasm.

Term

Ms. Take is a 24-year-old woman in the midst of an asthmatic attack. She has received multiple doses of bronchodilators but remains severely dyspneic. Arterial blood gases show a pH of 7.42, PCO2 of 30 and PO2 of 54 on 3 liters of supplemental oxygen.


 

8. Predict the effect of increasing her oxygen flow rate on her arterial PCO2? Why?

 

Definition

1. There are several possibilities depending on whether she can increase her minute ventilation to compensate for the increase in V/Q matching engendered by the additional oxygen.


2. Only partial credit was given for answers that did not address the issue of whether she was able to increase her minute ventilation sufficiently to keep her PCO2 constant despite the changes in V/Q matching.


3. Remember, supplemental oxygen alters V/Q matching in all patients.


4. Only those who cannot or “will not” have have a compensatory increase in minute ventilation will have an increase in PCO2.

Term

Ms. Take is a 24-year-old woman in the midst of an asthmatic attack. She has received multiple doses of bronchodilators but remains severely dyspneic. Arterial blood gases show a pH of 7.42, PCO2 of 30 and PO2 of 54 on 3 liters of supplemental oxygen.


9.  The patient states that it feels difficult for her to get the air into her lungs. Why?

Definition

1. Due to air trapping her end-expiratory volume is increased over FRC.

 

2. This places her lungs on a less compliant portion of their pressure volume curve and increases her inspiratory work of breathing.

Term

You are paged "stat" to see Mr. Reque, a 60 year old man with multiorgan system failure who is receiving paralyzing agents to facilitate ventilator management. The nurses hand you a blood gas result that shows a pH of 7.15, PCO2 of 40 and PO2 of 150 while breathing with at a ventilator rate of 20 with an FIO2 of 40%. Further blood tests reveal an elevated lactate level.


10.          What will happen to his PCO2 if he receives intravenous bicarbonate? Why?

 

 

Definition

1. His PCO2 will increase due to the increased CO2 production that occurs when:


2. Bicarbonate buffers the acid and


3. His inability to increase minute ventilation to compensate.

Term

You are paged "stat" to see Mr. Reque, a 60 year old man with multiorgan system failure who is receiving paralyzing agents to facilitate ventilator management. The nurses hand you a blood gas result that shows a pH of 7.15, PCO2 of 40 and PO2 of 150 while breathing with at a ventilator rate of 20 with an FIO2 of 40%. Further blood tests reveal an elevated lactate level.


 

11.          Describe his acid-base status.

 

Definition

He has a metabolic acidosis without respiratory compensation.

Term

You are reading pulmonary function tests as part of your pulmonary elective. Mr. Lannaec has a FVC of 2.3 liters (60% of predicted), and FEV1 of 1.1 liters (40% of predicted) with an FEV1/FVC ratio of 48%. His total lung capacity (TLC) is 6.2 liters (130% predicted). His DlCO is 40% of predicted.


12. What is the most likely cause of airflow obstruction in this person?

 

Definition

He has emphysema.

Term

You are reading pulmonary function tests as part of your pulmonary elective. Mr. Lannaec has a FVC of 2.3 liters (60% of predicted), and FEV1 of 1.1 liters (40% of predicted) with an FEV1/FVC ratio of 48%. His total lung capacity (TLC) is 6.2 liters (130% predicted). His DlCO is 40% of predicted.


 

13-14.    Give two reasons for the decrease in DlCO.

Definition

1. He has loss of capillary volume and of surface

area for diffusion.

 

2. There is not thickening or scarring of the alveolar

membrane in emphysema.

 

 

Term

You are reading pulmonary function tests as part of your pulmonary elective. Mr. Lannaec has a FVC of 2.3 liters (60% of predicted), and FEV1 of 1.1 liters (40% of predicted) with an FEV1/FVC ratio of 48%. His total lung capacity (TLC) is 6.2 liters (130% predicted). His DlCO is 40% of predicted.


15. Why is total lung capacity increased?

Definition

1. Total lung capacity is the point where the

inwardly directed recoil forces of the lung and

chest wall counterbalance the outwardly directed

forces of the inspiratory muscles.

 

2. The increase in lung compliance allows the

inspiratory muscles to expand the respiratory

system to a higher total lung capacity.

 

To get full credit you had to specifically mention the inspiratory muscles.

 

Term

You are reading pulmonary function tests as part of your pulmonary elective. Mr. Lannaec has a FVC of 2.3 liters (60% of predicted), and FEV1 of 1.1 liters (40% of predicted) with an FEV1/FVC ratio of 48%. His total lung capacity (TLC) is 6.2 liters (130% predicted). His DlCO is 40% of predicted.


16-17.    List two reasons why the increase in TLC adversely affects diaphragmatic function.

Definition

1. The diaphragms are shorter and can thus generate less tension.

 

2. They are flatter with a larger radius of curvature.

 

3. Thus, by LaPlace's Law, they are less efficient at generating pressure.

 

4. Finally, the vector of force when they contract can cause the lower rib cage to move in rather than expand during inspiration (Hoover's sign).

 

 

Term

You are called to evaluate a patient who is comatose following head trauma. He has been hyperventilated to reduce cerebral edema and has no spontaneous ventilation. Arterial blood gasses while receiving mechanical ventilation show a pH of 7.58, PCO2 of 20 and PO2 of 65 with an arterial oxygen saturation of 98%.


18-19.    List two ways a reduction in the ventilator rate will change his arterial oxygen saturation.

 

Definition

1. Due to the alveolar gas equation, the increased PCO2 caused by the decreased minute ventilation will decrease the arterial PO2. (Note: this is the same as saying that the decreased minute ventilation causes a decrease in the alveolar PO2).

 

2. The second reason is the shift in the oxyhemoglobin dissociation curve due to the decreased pH, which causes a decrease in O2 saturation for the same level of PO2.

Term

Mr. Wright is a 68-year-old former construction worker who is on disability because of extensive asbestosis. He is recovering from a broken hip when he develops severe dyspnea, pleuritic chest pain, and becomes hypotensive. He is successfully resuscitated. Pulmonary arteriography reveals a pulmonary embolus occluding a branch leading to the right lower lobe.


20.          Despite a relatively small embolus, Mr. Wright is hypotensive. Why?

 

Definition

1. His preexisting lung disease has destroyed capillary beds.


2. He is thus unable to recruit and dilate additional capillaries to:


3. Decrease his pulmonary artery resistance and


4. Compensate for a relatively small embolus.

Term

Ms. Terry is a 25-year-old woman with ARDS following a heroin overdose. She is receiving mechanical ventilation and a pulmonary artery catheter is in place. Arterial blood gases on an FIO2 of 50% show a pH of 7.39 PCO2 of 40 and PO2 of 70. Mixed venous PO2 is 34 with an oxygen saturation of 65%. Her PEEP is increased from 5 to 10 cm H2O. Repeat arterial blood gases on the new setting shows a pH of 7.39, PCO2 of 40 and PO2 of 80. Mixed venous oxygen PO2 is now 30 with an oxygen saturation of 60%.


21.          How can you explain the decrease in mixed venous PO2?

 

 

Definition

The increased level of PEEP had decreased:

1. Her cardiac output and thus

2. Her tissue oxygen delivery.


3. To compensate, her tissues must extract a greater percentage of the oxygen.

Term

Mr. Coe is a 45-year-old man who is a long distance runner who has competed in numerous marathons. His 45-year-old neighbor, Mr. Couch, is sedentary and gets most of his exercise changing television channels.


22. Compare the neighbors with respect to maximum achievable heart rates.

 

 

 

Definition

Maximum heart rate is determined by age and they are thus the same.

Term

Mr. Coe is a 45-year-old man who is a long distance runner who has competed in numerous marathons. His 45-year-old neighbor, Mr. Couch, is sedentary and gets most of his exercise changing television channels.

 

 

23.  Who will have a higher maximum oxygen consumption and why?

Definition

1. Mr. Coe is better able to increase his cardiac output and


2. His tissues are better able to extract oxygen.


3. He will have the higher maximum oxygen consumption.

Term

Mr. Starling is a 75 year old man who presents with increasing dyspnea, inability to lie flat, and episodes of sudden awakening at night (paroxysmal nocturnal dyspnea). Physical examination shows signs of congestive heart failure. A chest x-ray shows signs of congestive heart failure including a right pleural effusion.


24.          Is it likely that the protein concentration of the pleural fluid will be high or low? Why?

 

 

Definition

1. Congestive heart failure causes transudative effusions due to increased hydrostatic pressure within the pleural capillaries.


2. The protein is low because capillary permeability to proteins is unchanged.

Term

Mr. Starling is a 75 year old man who presents with increasing dyspnea, inability to lie flat, and episodes of sudden awakening at night (paroxysmal nocturnal dyspnea). Physical examination shows signs of congestive heart failure. A chest x-ray shows signs of congestive heart failure including a right pleural effusion.

 

 

25. What factors tend to decrease the fluid flux across the pleural membrane?

Definition

1. As the fluid accumulates, the hydrostatic pressure in the pleural space increases


2. This partially counterbalances the increased capillary pressure.


3. Additionally, the loss of fluid from the capillaries increases the oncotic pressure within them.

Term

You are asked for a medical consult on Mr. Rogers, a 69 year old man admitted after fracturing his right femur. The patient has not seen a physician for over 20 years. He is a retired truck driver with a 100 pack year smoking history (2 packs a day for 50 years). He denies any dyspnea on exertion but states that he has a sedentary life style. He admits to a chronic cough productive of 1 cup of sputum per day. Physical examination reveals an obese man in no acute distress. Auscultation of the lungs show diminished air entry, prolonged expiratory phase and scattered wheezes on forced exhalation. Hoover's sign is absent. Cardiac exam reveals signs of pulmonary hypertension. Arterial blood gases show a pH of 7.36, PCO2 of 50 and PO2 of 54 breathing room air. Pulmonary function tests show an FVC of 1.6 liters (40% of predicted) and an FEV1 of 0.8 liters (26% of predicted).


1-4. Is it likely that Mr. Rogers has predominantly emphysema or chronic bronchitis? Give three pieces of evidence to support your contention.

 

 

Definition

1. Mr. Rogers has predominantly chronic bronchitis.


2. Clinically, he has chronic cough and sputum production;


3. He is hypoxemic and retaining CO2


4. He does not have a Hoover's sign indicating that he is not hyper-expanded.

Term

You are asked for a medical consult on Mr. Rogers, a 69 year old man admitted after fracturing his right femur. The patient has not seen a physician for over 20 years. He is a retired truck driver with a 100 pack year smoking history (2 packs a day for 50 years). He denies any dyspnea on exertion but states that he has a sedentary life style. He admits to a chronic cough productive of 1 cup of sputum per day. Physical examination reveals an obese man in no acute distress. Auscultation of the lungs show diminished air entry, prolonged expiratory phase and scattered wheezes on forced exhalation. Hoover's sign is absent. Cardiac exam reveals signs of pulmonary hypertension. Arterial blood gases show a pH of 7.36, PCO2 of 50 and PO2 of 54 breathing room air. Pulmonary function tests show an FVC of 1.6 liters (40% of predicted) and an FEV1 of 0.8 liters (26% of predicted).


5. Why does Mr. Rogers have pulmonary hypertension?

Definition

He has:

1. Chronic hypoxemia and

2. Hypercapnia


3. Which cause pulmonary vasoconstriction

Term

Ms. Suris is a 49 year old woman who presented with severe gallstone-induced pancreatitis leading to the adult respiratory distress syndrome. She had to be paralyzed to facilitate intubation and mechanical ventilation. She steadily improved over time and her physicians had been able to decrease the inspired oxygen level to 40%. Arterial blood gases at that point were pH of 7.40 PCO2 of 40 and PO2 of 100. Prior to the physicians discontinuing her paralysis she became acutely hypotensive (60/40) and tachycardic. Arterial blood gases on an FIO2 of 100% showed a pH of 7.10 PCO2 of 65 and PO2 of 60. Cardiac output measured using a pulmonary artery (Swan-Ganz) catheter was 1.5 L/min (normal >5 L/min). Shortly thereafter, the patient suffered a cardiopulmonary arrest and could not be resuscitated. Postmortem examination showed a large "saddle" embolus occluding the main pulmonary artery.


6.            Why did Ms. Suris become hypotensive after her pulmonary embolus?

 

 

Definition

1. The large embolus blocked flow through the pulmonary arteries


2. This impeded filling of the left ventricle, as the right ventricle could not acutely generate sufficient enough pressure to generate flow.

Term

Ms. Suris is a 49 year old woman who presented with severe gallstone-induced pancreatitis leading to the adult respiratory distress syndrome. She had to be paralyzed to facilitate intubation and mechanical ventilation. She steadily improved over time and her physicians had been able to decrease the inspired oxygen level to 40%. Arterial blood gases at that point were pH of 7.40 PCO2 of 40 and PO2 of 100. Prior to the physicians discontinuing her paralysis she became acutely hypotensive (60/40) and tachycardic. Arterial blood gases on an FIO2 of 100% showed a pH of 7.10 PCO2 of 65 and PO2 of 60. Cardiac output measured using a pulmonary artery (Swan-Ganz) catheter was 1.5 L/min (normal >5 L/min). Shortly thereafter, the patient suffered a cardiopulmonary arrest and could not be resuscitated. Postmortem examination showed a large "saddle" embolus occluding the main pulmonary artery.


7-8.        Give two factors contributing to her elevated PCO2.

 

Definition

1. She had a marked increase in dead space from the embolus


2. However, she was paralyzed and thus unable to increase her minute ventilation to compensate.


3. The decrease in pH was greater than expected given the increase in her PCO2, indicating a concomitant metabolic acidosis.


4. The buffering of the hydrogen ion causes generation of CO2.


5. Again, since she is paralyzed, she cannot increase her minute ventilation to compensate.

Term

Ms. Suris is a 49 year old woman who presented with severe gallstone-induced pancreatitis leading to the adult respiratory distress syndrome. She had to be paralyzed to facilitate intubation and mechanical ventilation. She steadily improved over time and her physicians had been able to decrease the inspired oxygen level to 40%. Arterial blood gases at that point were pH of 7.40 PCO2 of 40 and PO2 of 100. Prior to the physicians discontinuing her paralysis she became acutely hypotensive (60/40) and tachycardic. Arterial blood gases on an FIO2 of 100% showed a pH of 7.10 PCO2 of 65 and PO2 of 60. Cardiac output measured using a pulmonary artery (Swan-Ganz) catheter was 1.5 L/min (normal >5 L/min). Shortly thereafter, the patient suffered a cardiopulmonary arrest and could not be resuscitated. Postmortem examination showed a large "saddle" embolus occluding the main pulmonary artery.

 

9.            Ms. Suris' mixed venous oxygen tension was low when she was hypotensive. Why?

Definition

Her mixed venous oxygen tension will decrease due to:


1. Decreased tissue oxygen delivery secondary to the


2. Decreased cardiac output.

Term

Mr. Candy is an 18 year old man who ran out of his insulin two days ago. He was admitted to the intensive care unit yesterday with diabetic ketoacidosis. On admission arterial blood gases were pH of 7.20, PCO2 of 18 and PO2 was 110 breathing room air. He has responded well to hydration and intravenous insulin therapy. Arterial blood gases today show pH of 7.48, PCO2 of 30 and PO2 of 95.


10.  Why does Mr. Candy still have a decreased PCO2 on his second arterial blood gas?

 

 

Definition

1. The CNS chemoreceptors respond to the pH of the CSF.


2. The blood brain barrier is relatively impermeable to bicarbonate but freely permeable to CO2.


3. Thus the pH of the CSF will lag behind that of the arterial blood.


4. The CNS receptors will therefore continue to increase minute ventilation.

Term

Mr. Candy is an 18 year old man who ran out of his insulin two days ago. He was admitted to the intensive care unit yesterday with diabetic ketoacidosis. On admission arterial blood gases were pH of 7.20, PCO2 of 18 and PO2 was 110 breathing room air. He has responded well to hydration and intravenous insulin therapy. Arterial blood gases today show pH of 7.48, PCO2 of 30 and PO2 of 95.

 

11. Why did his arterial PO2 decrease on his second arterial blood gas?

Definition

1. There is an increase in the PCO2.


2. The A-a gradient remains normal.

Term

Mr. Coe is a 16 year old high school student hoping to make the track team who comes to you with several questions. Please answer them.


12.  What types of exercise can I do to increase my maximum heart rate?

 

 

Definition

1. There is no exercise that will increase the maximum heart rate


2. It is determined by age.

Term

Mr. Coe is a 16 year old high school student hoping to make the track team who comes to you with several questions. Please answer them.


13.  I read in my older sister's pulmonary pathophysiology syllabus that retaining CO2 decreases the work of breathing by reducing the minute ventilation required to blow off the usual amount of CO2 produced by the body. Would breathing in extra CO2 during exercise decrease my minute ventilation? Why or why not?

Definition

1. Rebreathing CO2 will not decrease your minute ventilation because:


2. The rise in CO2 will increase minute ventilation.

Term

Mr. Reeve is a 38 year old man who suffered a complete transection of the spinal cord at the C6 level in a diving accident.


14. Why does he have difficulty generating an effective cough

 

Definition

He has lost the use of his abdominal muscles that

are needed for active exhalation

Term

Mr. Reeve is a 38 year old man who suffered a complete transection of the spinal cord at the C6 level in a diving accident.


15.   His pulmonary function tests have a decreased total lung capacity. Why?

Definition

He has lost the use of his accessory muscles of

inspiration

Term

Mr. Reeve is a 38 year old man who suffered a complete transection of the spinal cord at the C6 level in a diving accident.


16. His pulmonary function tests have a decrease in the FEV1/FVC ratio. Why?

Definition

He cannot exhale forcefully due to loss of the

abdominal musculature.

 

Term

Ms. Terry is a 20 year old asthmatic who has had increasing cough and sputum production over the past week following onset of a "cold". She finally presents to the Emergency Room stating that she is very short of breath. Physical examination reveals a respiratory rate of 30 with use of accessory muscles. There is a pulsus paradox of 20. Auscultation of the lungs reveal diffuse wheezes. She receives two treatments with nebulized beta-agonist agents and states that she feels better and has to go home to feed her family. She returns to the Emergency Room that night in respiratory failure and requires intubation.


17.    What test should have been done to best assess Ms.Terry's response to therapy during her first visit?

 

Definition

1. Pulmonary function testing (i.e., spirometry, measurement of forced vital capacity or FEV1, or peak expiratory flow).

 

Term

Ms. Terry is a 20 year old asthmatic who has had increasing cough and sputum production over the past week following onset of a "cold". She finally presents to the Emergency Room stating that she is very short of breath. Physical examination reveals a respiratory rate of 30 with use of accessory muscles. There is a pulsus paradox of 20. Auscultation of the lungs reveal diffuse wheezes. She receives two treatments with nebulized beta-agonist agents and states that she feels better and has to go home to feed her family. She returns to the Emergency Room that night in respiratory failure and requires intubation.


18.  Why might there have been continued airway obstruction despite therapy with bronchodilators?

 

Definition

1. Airway inflammation and


2. Mucous hypersecretion.

Term

Mr. Manville was a pipe fitter for 30 years during which time he was exposed to large amounts of environmental asbestos particles. He now has interstitial lung disease and is severely dyspneic on minimal exertion.


19. Despite low values for forced vital capacity (FVC) and forced expiratory volume in one second (FEV1) his FEV1/FVC ratio is preserved. Why?

 

Definition

1. He does not have airway obstruction.

 

2. Thus the amount of air that is contained in his lungs is decreased but:

 

3. He is able to exhale what air he has normally.

Term

Mr. Manville was a pipe fitter for 30 years during which time he was exposed to large amounts of environmental asbestos particles. He now has interstitial lung disease and is severely dyspneic on minimal exertion.

 

 

20-21.    Give two reasons for the decrease in his DlCO.

Definition

1. He has thickened membranes and:

 

2. Loss of capillary blood volume.

 

 

 

Term

Mr. Manville was a pipe fitter for 30 years during which time he was exposed to large amounts of environmental asbestos particles. He now has interstitial lung disease and is severely dyspneic on minimal exertion.


22.  Why is carbon monoxide used to measure diffusing capacity?

Definition

1. Carbon monoxide is strongly bound to hemoglobin so the "back pressure" (P2 in the equation) impeding diffusion is minimal (DM is proportional to A/T * D (P1 - P2)).

Term

Mr. Claus is an elderly man who is morbidly obese. Arterial blood gases show a pH of 7.36, PCO2 of 64 and PO2 of 58.


23. Why is he hypoxemic?

Definition

1. His PCO2 is elevated due to

hypoventilation.

Term

Mr. Claus is an elderly man who is morbidly obese. Arterial blood gases show a pH of 7.36, PCO2 of 64 and PO2 of 58.

 

24. Is it likely that he chronically retains CO2. Why?

Definition

1. It is likely chronic.

 

2. The decrease in pH is less than expected for the increase in PCO2

 

3. This indicates that he has had time to achieve metabolic compensation for the respiratory acidosis by retaining bicarbonate.

 

Term

Mr. Claus is an elderly man who is morbidly obese. Arterial blood gases show a pH of 7.36, PCO2 of 64 and PO2 of 58.

 

25. When asked to voluntarily hyperventilate, his PCO2 does not decrease. Why?

Definition

1. (Written by a student on the old exam) ↑in ventilation ↑work of breathing ↑production of CO2 by muscle matches the CO2 blown off

Term

Mr. Baron is a 34 year old man who develops ARDS following an automobile accident in which he sustains multiple injuries. A pulmonary artery catheter is placed to aid in management. The following measurements are obtained over the next three hours.


 

PEEP (cm H2O)

Arterial blood PO2

Arterial oxygen saturation (%)

Cardiac output

0

45

72

4.0

5

50

78

3.6

10

65

90

3.0

15

85

95

2.8

 

 

1-2.        Which level of PEEP is optimal and why?

 

Definition

The PEEP of 10 optimizes tissue oxygen delivery.

 

From WUMSWeb:

No one can figure out the answer to 1 and 2.

Here is the logic: you want to maximize O2 delivery = (cardiac output)x(O2 content)

O2 content = 1.34 * Hgb * (O2 sat) + 0.003*PO2, but the PO2 part is pretty negligible

O2 delivery = 1.34 * Hgb * (O2 sat) * (cardiac output)

If you run the numbers (assuming normal Hgb), you don't get that 10 mmHg maximizes the value.

Lippman says you need to know the hemoglobin concentrations to answer that (this isn't true because it cancels out). He also said the PO2 is useless to know since the PO2 of the cell is incredibly low and there is always a gradient.

Thanks to Jon Powers for bringing this up and talking to Lippman.

 

Term

Mr. Green is a 79 year old man with increasing dyspnea on exertion. Pulmonary function tests reveal a forced vital capacity (FVC) of 1.8 liters and a forced expiratory volume in one second (FEV1) of 1.6 liters.


3.  Will his measured total lung capacity likely be high, low, or normal? Why?

Definition

1. Spirometry is compatible with a restrictive

process

 

2. The TLC should be low.

Term

Mr. Green is a 79 year old man with increasing dyspnea on exertion. Pulmonary function tests reveal a forced vital capacity (FVC) of 1.8 liters and a forced expiratory volume in one second (FEV1) of 1.6 liters.


4.   Will his functional residual capacity be high, low, or normal? Why?

Definition

1. The FRC in restrictive processes is lower than normal due to the decreased lung compliance

 

2. This places the resting volume of the lung and chest wall at a lower level.

Term

Mr. Green is a 79 year old man with increasing dyspnea on exertion. Pulmonary function tests reveal a forced vital capacity (FVC) of 1.8 liters and a forced expiratory volume in one second (FEV1) of 1.6 liters.

 

5.  Why is his vital capacity decreased?

Definition

1. In restrictive diseases, the decrease in TLC is

greater than any decrease in residual volume.

Term

Mr. Gregory is a 17 year old man who suffered a C6 spinal cord transaction from a horseback riding accident.


6.  Why is his total lung capacity decreased?

 

Definition

1. He has lost the use of his external intercostal muscles

 

2. These are the muscles which contract during inspiration and keep the intercostal spaces from collapsing inward.

 

3. Additionally, loss of muscle tone in his abdominal musculature increases abdominal compliance.

 

4. This decreases the amount of expansion of the chest wall during diaphragmatic contraction

 

5. This is because the abdominal contents will not provide as stable a fulcrum to separate the rib cage.

 

Term

Mr. Gregory is a 17 year old man who suffered a C6 spinal cord transaction from a horseback riding accident.

 

7. Why is his residual volume increased?

 

Definition

1. The muscles that cause active expiration (primarily the abdominal muscles) are paralyzed.

Term

Mr. Gregory is a 17 year old man who suffered a C6 spinal cord transaction from a horseback riding accident.

 

 

8. Why doesn't Mr. Gregory have respiratory paradox?

Definition

1. Respiratory paradox occurs when the diaphragms are paralyzed.

 

2. His are not as the transection level spares the diaphragms.

Term

Mrs. Cochran is a 60 year old woman with emphysema. On physical examination she has a barrel chest. Heart sounds are distant. There are no wheezes. Hoover's sign is present. Chest radiographs reveal a low flat diaphragm.

 

 

12. Why does Mrs. Cochran have more dyspnea climbing stairs if she is carrying a load of laundry?

 

Definition

1. Low, flat diaphragms are shorter so they develop less tension

 

2. They are flatter so they are less able to transform the tension into pressure (LaPlace's law)

 

3. They have lost the area of apposition to the chest wall

 

4. As such, the vector of force when they contract actually decreases the size of the thoracic cavity by pulling in on the lower rib cage (Hoover's sign)

 

 

Term

Mrs. Cochran is a 60 year old woman with emphysema. On physical examination she has a barrel chest. Heart sounds are distant. There are no wheezes. Hoover's sign is present. Chest radiographs reveal a low flat diaphragm.

 

 

12. Why does Mrs. Cochran have more dyspnea climbing stairs if she is carrying a load of laundry?

 

Definition

1. She is using her accessory muscles of ventilation to do the lifting.

 

2. They are thus unable to augment the inspiratory effort of the diaphragm

 

3. The diaphragm therefore has to generate a greater percentage of its maximum pressure for a given tidal volume.

Term

Ms. Johnson is a previously healthy 24 year old woman whose leg has been in a cast for the past five weeks. She suddenly develops right-sided pleuritic chest pain accompanied by dyspnea and blood-streaked sputum. A ventilation perfusion scan reveals absent perfusion with normal ventilation to the right upper lobe. Arterial blood gases on presentation are pH 7.48 PCO2 30 and PO2 of 76 while breathing 35% oxygen by face mask.


13.          Describe Ms. Johnson's acid-base status.

Definition

1. She has a respiratory alkalosis

 

2. Her PCO2 is decreased and the pH is elevated.

 

 

 

Term

Ms. Johnson is a previously healthy 24 year old woman whose leg has been in a cast for the past five weeks. She suddenly develops right-sided pleuritic chest pain accompanied by dyspnea and blood-streaked sputum. A ventilation perfusion scan reveals absent perfusion with normal ventilation to the right upper lobe. Arterial blood gases on presentation are pH 7.48 PCO2 30 and PO2 of 76 while breathing 35% oxygen by face mask.


14. Are the changes likely to be acute?

 

 

Definition

1. The increase in pH is in the expected proportion to the decrease in PCO2

 

2. This indicates that the changes are likely acute.

Term

Ms. Johnson is a previously healthy 24 year old woman whose leg has been in a cast for the past five weeks. She suddenly develops right-sided pleuritic chest pain accompanied by dyspnea and blood-streaked sputum. A ventilation perfusion scan reveals absent perfusion with normal ventilation to the right upper lobe. Arterial blood gases on presentation are pH 7.48 PCO2 30 and PO2 of 76 while breathing 35% oxygen by face mask.

 

15. Ms. Johnson's PCO2 is low despite a normal PO2. Why?

Definition

1. Release of mediators from the embolus has stimulated irritant receptors within the lung

 

2. This causes alveolar hyperventilation.

Term

Despite their age differences, Mr. Shulman (who is 40 years old) and Mr. Harmon (65 years old) enjoy swimming competitively and are in excellent physical condition each achieving a maximal oxygen consumption of 60 ml of oxygen/kg.


16. Which swimmer has the higher stroke volume at maximal exercise? Why?

 

 

Definition

1. Mr. Harmon is older and thus has a lower maximum heart rate.

 

2.  In order for him to have the same maximal oxygen consumption as Mr. Shulman, Mr. Harmon must have a higher stroke volume.

 

Term

Despite their age differences, Mr. Shulman (who is 40 years old) and Mr. Harmon (65 years old) enjoy swimming competitively and are in excellent physical condition each achieving a maximal oxygen consumption of 60 ml of oxygen/kg.

 

17.          Can you tell from the data who is the faster swimmer? Why?

 

Definition

1. No.

 

2. Speed is determined not only by maximal oxygen consumption, but also by stroke efficiency, buoyancy, resistance, etc. (A Porsche has less horsepower than a tank but goes considerably faster).

Term

Ms. Smith is in the midst of an asthma exacerbation. Arterial blood gases show a pH of 7.42, a PCO2 of 30 and a PO2 of 62 on room air.


18. Is it likely that the attack just started? Why?

 

19. What will happen to her PCO2 when she receives supplemental oxygen? Why?

 

20-22.    List three findings (qualitative) you would expect on her pulmonary function tests during this exacerbation.

 

Definition

1. The attack is not likely to have just started.

 

2. A PCO2 of 30 should cause the pH to increase to 7.48.

 

3. The fact that the pH is only 7.42 indicates that the body has had time to compensate for the respiratory alkalosis by excreting bicarbonate.

 

Term

Ms. Smith is in the midst of an asthma exacerbation. Arterial blood gases show a pH of 7.42, a PCO2 of 30 and a PO2 of 62 on room air.

 

 

19. What will happen to her PCO2 when she receives supplemental oxygen? Why?

Definition

1. It probably will not change

 

2. She should have a normal ventilatory drive and be able to increase her minute ventilation to compensate for the changes that occur with administration of supplemental oxygen.

Term

Ms. Smith is in the midst of an asthma exacerbation. Arterial blood gases show a pH of 7.42, a PCO2 of 30 and a PO2 of 62 on room air.

 

20-22.    List three findings (qualitative) you would expect on her pulmonary function tests during

this exacerbation.

Definition

1. She should have PFTs compatible with airway obstruction.

 

2. Thus her FVC and FEV1 should be decreased

 

3.  As should the FEV1/FVC ratio.

Term

Mr. Edwards is an obese 43 year old man whose wife states that she is about ready to divorce him as he is chronically sleepy and is making her chronically sleepy with his snoring. Examination reveals signs of pulmonary hypertension.


23. What is the likely diagnosis in this patient?

Definition

Obstructive sleep apnea.

Term

Mr. Edwards is an obese 43 year old man whose wife states that she is about ready to divorce him as he is chronically sleepy and is making her chronically sleepy with his snoring. Examination reveals signs of pulmonary hypertension.

 

24. What causes the pulmonary hypertension in this disorder?

Definition

Recurring episodes of hypoxemia.

Term

Mr. Edwards is an obese 43 year old man whose wife states that she is about ready to divorce him as he is chronically sleepy and is making her chronically sleepy with his snoring. Examination reveals signs of pulmonary hypertension.

 

 

 

25.   Why is he chronically sleepy?

Definition

He has multiple arousals from sleep during the night

due to hypoxemia.

Term

Ms. Brown is a previously healthy, obese 24 year old female on birth control pills who developed severe dyspnea, pleuritic chest pain and hypotension three days after returning from a long car trip. A pulmonary angiogram revealed a pulmonary embolus.


1. The embolus is almost certain to be very large. Why?

Definition

1. She is hypotensive and thus has lost a large proportion of her pulmonary circulation (>50%)

 

2. As such, her right ventricle is unable to generate sufficient pressure to maintain cardiac output to the left ventricle.

 

Term

Ms. Brown is a previously healthy, obese 24 year old female on birth control pills who developed severe dyspnea, pleuritic chest pain and hypotension three days after returning from a long car trip. A pulmonary angiogram revealed a pulmonary embolus.

 

 

2. Why is it unlikely that she will have a mean right ventricular pressure greater than 40 mm Hg?

 

Definition

The right ventricle is unable to generate pressures

greater than 40 mmHg acutely.

Term

Mr. Jones is a 45 year old man who developed bilateral paralysis of his diaphragms following injury to his phrenic nerves during cardiac surgery.


3.  Why is his total lung capacity decreased?

 

 

Definition

1. He can only inhale using his accessory muscles

 

2. These muscles are not as efficient in expanding the thoracic cavity.

 

Term

Mr. Jones is a 45 year old man who developed bilateral paralysis of his diaphragms following injury to his phrenic nerves during cardiac surgery.

 

4.  Although he can usually walk upstairs from the basement he is unable to carry laundry up the steps. Why?

Definition

1. When he carries objects up the steps his accessory muscles of inspiration are being used to support the weight of the objects being carried

 

2. As such, these muscles are thus less efficient at moving the chest wall.

Term

Mr. White is a 75 year old white male with severe airway obstruction due to chronic bronchitis. Three days prior to admission he developed a cough productive of green sputum, fever and progressive dyspnea. On presentation to the Emergency Department he was confused and cyanotic. Arterial blood gases drawn on room air revealed a pH of 7.16, PCO2 of 70 and PO2 of 30 and a calculated bicarbonate of 24.


5.   Why is he acidotic?

Definition

1. The pH is decreased in the expected proportion to the increase in PCO2.

 

2. He has an acute respiratory acidosis.

 

 

Term

Mr. White is a 75 year old white male with severe airway obstruction due to chronic bronchitis. Three days prior to admission he developed a cough productive of green sputum, fever and progressive dyspnea. On presentation to the Emergency Department he was confused and cyanotic. Arterial blood gases drawn on room air revealed a pH of 7.16, PCO2 of 70 and PO2 of 30 and a calculated bicarbonate of 24.


6-7.  On the basis of the initial arterial blood gases, is it likely that Mr. White chronically retained carbon dioxide? Why?

 

Definition

1. He has not had time to retain bicarbonate to buffer the increased PCO2.

 

2. It is an acute change.

Term

Ms. Kelly is a 20 year old woman who suffered severe head trauma in an auto accident and is now comatose. She requires mechanical ventilation as she has no spontaneous ventilation. She had been medically stable for the past week. Arterial blood gases drawn yesterday while she was receiving an FIO2 of 35% were pH 7.44, PCO2 35, PO2 of 200. Today, she developed a fever to 104°. Arterial blood gases drawn on the same ventilator, settings are pH 7.28, PCO2 55, PO2 70. Chest radiographs show an extensive pneumonia.


8-9.        List two reasons for the increase in arterial PCO2.

 

 

 

Definition

1. She has a fever that increases her CO2 production.

 

2. Since she is paralyzed she cannot increase her minute ventilation to compensate.

 

3. She also cannot increase her minute ventilation to compensate for the increased V/Q mismatch caused by her pneumonia.

 

 

Term

Ms. Kelly is a 20 year old woman who suffered severe head trauma in an auto accident and is now comatose. She requires mechanical ventilation as she has no spontaneous ventilation. She had been medically stable for the past week. Arterial blood gases drawn yesterday while she was receiving an FIO2 of 35% were pH 7.44, PCO2 35, PO2 of 200. Today, she developed a fever to 104°. Arterial blood gases drawn on the same ventilator, settings are pH 7.28, PCO2 55, PO2 70. Chest radiographs show an extensive pneumonia.

 

10-11.    List two reasons for the decrease in arterial PO2.

 

Definition

1. She has V/Q mismatch due to the pneumonia

 

2. The elevation in PCO2 decreases her PO2 (alveolar gas equation).

Term

Mr. Harris is a 45 year old executive whose best friend just had a myocardial infarction. Mr. Harris has decided to give up smoking (he has smoked two packs per day for the past 28 years) and to join an exercise program. Despite a lack of improvement in his moderate airflow obstruction he increased his maximal oxygen consumption by over 25% in the course of one year.


12-13.    List two physiologic changes secondary to training which enabled him to increase his maximum oxygen consumption.

 

Definition

1. Exercise has enabled Mr. Harris to increase his cardiac output by increasing his stroke volume.

 

2. It has also increased the number of capillaries in his muscles

 

3. It has also increased their number of mitochondria and altered the respiratory enzymes within the mitochondria making them bind oxygen more tightly.

 

4. The effects of the latter changes allow him to decrease his mixed venous oxygen further than in the untrained state.

 

Term

Mr. Gilbert is a 65 year old man who worked with asbestos insulation for over 30 years before retiring 15 years ago. He is now dyspneic on walking 100 yards. Chest radiographs show a marked increase in interstitial markings and pulmonary function tests reveal a marked restrictive pattern.


14.          Why is the total lung capacity reduced in interstitial lung diseases?

 

 

Definition

The decreased lung compliance makes it more difficult for the inspiratory muscles to fully expand the lungs.

 

Term

Mr. Gilbert is a 65 year old man who worked with asbestos insulation for over 30 years before retiring 15 years ago. He is now dyspneic on walking 100 yards. Chest radiographs show a marked increase in interstitial markings and pulmonary function tests reveal a marked restrictive pattern.

 

 

15. Why is vital capacity decreased?

Definition

The decrease in TLC is greater than any decrease in residual volume.

Term

Mr. Gilbert is a 65 year old man who worked with asbestos insulation for over 30 years before retiring 15 years ago. He is now dyspneic on walking 100 yards. Chest radiographs show a marked increase in interstitial markings and pulmonary function tests reveal a marked restrictive pattern.

 

 

16.  Why is the ratio of FEV1/FVC preserved in interstitial lung diseases?

 

Definition

1. The airways are normal.

 

2. Thus the air that is within the lungs can be exhaled at the normal rate.

 

3. There is just less air to exhale.

Term

Mr. Carson is a 35 year old man with who presents with hypotension, and peritonitis. He remains hypotensive following removal of a ruptured appendix and a pulmonary artery catheter is placed to help with his management. Arterial blood gases are pH of 7.25, PCO2 25, and PO2 of 60 while on mechanical ventilation with an FIO2 of 1. A chest radiograph is compatible with the adult respiratory distress syndrome (ARDS). Positive end expiratory pressure (PEEP) is instituted at a level of 10 cm H2O and his arterial PO2 increases to 100.


17.  Why does PEEP increase arterial PO2 in ARDS?

Definition

It recruits closed lung units allowing them to participate in gas exchange (decreases shunt).

 

Term

Mr. Carson is a 35 year old man with who presents with hypotension, and peritonitis. He remains hypotensive following removal of a ruptured appendix and a pulmonary artery catheter is placed to help with his management. Arterial blood gases are pH of 7.25, PCO2 25, and PO2 of 60 while on mechanical ventilation with an FIO2 of 1. A chest radiograph is compatible with the adult respiratory distress syndrome (ARDS). Positive end expiratory pressure (PEEP) is instituted at a level of 10 cm H2O and his arterial PO2 increases to 100.


18. How can we determine whether PEEP increased tissue oxygen delivery?

Definition

By measuring mixed venous oxygen saturation which reflects the balance between tissue oxygen delivery and demand.

Term

Mr. Carson is a 35 year old man with who presents with hypotension, and peritonitis. He remains hypotensive following removal of a ruptured appendix and a pulmonary artery catheter is placed to help with his management. Arterial blood gases are pH of 7.25, PCO2 25, and PO2 of 60 while on mechanical ventilation with an FIO2 of 1. A chest radiograph is compatible with the adult respiratory distress syndrome (ARDS). Positive end expiratory pressure (PEEP) is instituted at a level of 10 cm H2O and his arterial PO2 increases to 100.


19. Why doesn't PEEP improve oxygenation in patients with V /Q mismatch due to emphysema?

 

Definition

1. Patients with emphysema do not have alveoli that are collapsed.

 

2. Indeed, PEEP may worsen V/Q matching by raising alveolar pressure and decreasing blood flow to ventilated alveoli

 

3. This in turn shifts West zone 3 to zones 1 or 2.

Term

Mr. Gordon is a 68 year old man with severe airflow obstruction secondary to emphysema. He is severely dyspneic at rest. Arterial blood gases drawn while he is breathing room air at rest are pH 7.40, PCO2 40, PO2 62.


20.  Why is it unlikely that supplemental oxygen will improve his symptoms?

 

Definition

1. His PO2 is not at the level usually associated with increased ventilatory drive so his minute ventilation will not decrease.

 

2. Indeed, it may have to increase to compensate for the physiologic changes that occur with supplemental oxygen.

 

 

Term

Mr. Gordon is a 68 year old man with severe airflow obstruction secondary to emphysema. He is severely dyspneic at rest. Arterial blood gases drawn while he is breathing room air at rest are pH 7.40, PCO2 40, PO2 62.


21-22.    List two reasons why his arterial oxygen tension (PaO2) will decrease with exercise?

 

 

Definition

1. He may have decreased transit time through his capillaries leaving insufficient time for oxygen to equilibrate across the alveolo-capillary membrane.

 

2. He may develop pulmonary hypertension as he cannot recruit or dilate pulmonary capillaries to decrease pulmonary vascular resistance.

 

3. The decrease in mixed venous oxygen in the face of substantial V/Q mismatch can lower the arterial PO2.

Term

Mr. Gordon is a 68 year old man with severe airflow obstruction secondary to emphysema. He is severely dyspneic at rest. Arterial blood gases drawn while he is breathing room air at rest are pH 7.40, PCO2 40, PO2 62.


23.   Why does chronic CO2 retention help decrease dyspnea?

 

Definition

1. He has can exhale more CO2 per breath.

 

2. He thus can excrete the amount of CO2 produced each minute with lower minute ventilation.

Term

Ms. Gibson is a 17 year old woman with life-long asthma who has just started college. Her roommate receives a bouquet of roses (to which Ms. Gibson is highly allergic) and Ms. Gibson develops wheezing.


24-25.    Give two reasons why Ms. Gibson has a marked increase in minute ventilation following the onset of her asthma attack.

 

Definition

1. She has release of "mediators" which stimulate the irritant receptors in her lung parenchyma

 

2. This in turn increases her minute ventilation.

 

3. She has increased V/Q  mismatch

 

4. Thus she requires a higher minute ventilation to be able to maintain a normal PCO2.

 

Term

Mr. Cox is a 70 year old man who developed respiratory failure as a result of sepsis-induced adult respiratory distress syndrome. He required institution of mechanical ventilation with positive end expiratory pressure (PEEP) but still has a large intrapulmonary right to-left shunt. He was paralyzed to facilitate mechanical ventilation and thus has no spontaneous ventilation.


1-2.        What will happen to his arterial PCO2 if you double the tidal volume he receives from the ventilator but halve the respiratory rate? Why?

 

Definition

1. It will decrease because of a decrease in the dead space to tidal volume ratio (VD/VT).

 

2. Alveolar ventilation will therefore increase despite no net change in minute ventilation.

 

Term

Mr. Cox is a 70 year old man who developed respiratory failure as a result of sepsis-induced adult respiratory distress syndrome. He required institution of mechanical ventilation with positive end expiratory pressure (PEEP) but still has a large intrapulmonary right to-left shunt. He was paralyzed to facilitate mechanical ventilation and thus has no spontaneous ventilation.


3-4.        What will happen to his arterial pH if he develops a fever? Why?

Definition

1. His pH will decrease.

 

2. The fever increases his CO2 production.

 

3. Since he is paralyzed he will not be able to increase his minute ventilation to compensate.

 

Term

Mr. Cox is a 70 year old man who developed respiratory failure as a result of sepsis-induced adult respiratory distress syndrome. He required institution of mechanical ventilation with positive end expiratory pressure (PEEP) but still has a large intrapulmonary right to-left shunt. He was paralyzed to facilitate mechanical ventilation and thus has no spontaneous ventilation.

 

5-6.        What will happen to his arterial PO2 if he develops gastrointestinal bleeding with a large drop in his hemoglobin? Why?

Definition

1. The arterial PO2 will drop

 

2. This occurs because it is likely that tissue oxygen delivery will decrease and cause a drop in his mixed venous oxygen tension.

 

3. Since he has a large shunt this will decrease the arterial oxygen tension.

 

Term

Mrs. Allen is a 50 year old woman with increasing dyspnea over the past two years. Pulmonary function tests reveal a marked decrease in total lung capacity.


7-9.        Predict what you would see in three other tests of pulmonary function (flows or lung volumes).

 

Definition

1. Decreased FVC, FEV1

 

2. Normal FEV1/FVC ratio

 

3. Decreased vital capacity

 

4. Normal to decreased RV

 

5. Decreased FRC

 

Term

Mr. Ironman is an 80 year old marathon runner who has a maximal oxygen consumption twice as high (at least) as a 40 year old pulmonary physician.


10-11.    Give two adaptations that occur with training that allows this to happen.

 

Definition

1. He has a marked increase in his stroke volume.

 

2. He has peripheral adaptation in his muscles

 

3. This adaptation allows him to decrease his mixed venous oxygen tension to a greater degree than a sedentary person does.

 

 

Term

Mr. Brown is a 69 year old man with severe chronic obstructive pulmonary disease who develops chest pain, dyspnea, and severe hypotension one week after fracturing his hip. A pulmonary angiogram shows a pulmonary embolus occluding only 10% of the pulmonary circulation. A pulmonary artery catheter shows a mean pulmonary artery pressure of 40.


12.          Despite a relatively small pulmonary embolus Mr. Brown has elevated pulmonary artery pressures. Why?

Definition

He has preexisting lung disease, which has already impaired the ability of the pulmonary vascular bed to dilate and recruit new capillaries.

 

Term

Mr. Brown is a 69 year old man with severe chronic obstructive pulmonary disease who develops chest pain, dyspnea, and severe hypotension one week after fracturing his hip. A pulmonary angiogram shows a pulmonary embolus occluding only 10% of the pulmonary circulation. A pulmonary artery catheter shows a mean pulmonary artery pressure of 40.


13.          Why is he hypotensive?

Definition

1. His right ventricle is unable to generate sufficient pressure to maintain cardiac output to the left ventricle.

 

2. The LV therefore has insufficient preload to maintain adequate cardiac output.

 

Term

Ms. Lewis is a 20 year old who presents to the Emergency Room with an acute asthma attack. Her arterial blood gases are pH 7.48, PCO2 30, PO2 65 on room air. Mr. Green is in the next room with worsening of his chronic bronchitis. His arterial blood gases on room air are pH 7.35, PCO2 65, PO2 40.


14-15.    Whose changes in arterial PCO2 are acute? How can you tell?

 

 

Definition

Ms. Lewis who has the expected proportional decrease in her pH for the rise in PCO2.

 

Term

Ms. Lewis is a 20 year old who presents to the Emergency Room with an acute asthma attack. Her arterial blood gases are pH 7.48, PCO2 30, PO2 65 on room air. Mr. Green is in the next room with worsening of his chronic bronchitis. His arterial blood gases on room air are pH 7.35, PCO2 65, PO2 40.


16-17.    What will likely happen to Ms. Lewis' PCO2 if she gets supplemental oxygen? Why?

 

Definition

It will likely remain the same as she has a normal respiratory drive (unless her respiratory muscles are unable to increase minute ventilation due to fatigue).

 

Term

Mr. Marks is an 18 year old man with bilateral paralysis of his diaphragms. During exhalation he uses his abdominal muscles to exhale below functional residual capacity (FRC).


18.  Why is this pattern of breathing beneficial?

Definition

1. The recoil properties of his chest wall facilitate the next inhalation as it springs outward to the normal FRC position.

 

 

Term

Mr. Marks is an 18 year old man with bilateral paralysis of his diaphragms. During exhalation he uses his abdominal muscles to exhale below functional residual capacity (FRC).


19-20.    During inspiration the abdominal wall generally moves out as the chest wall expands. How will the abdominal wall move when Mr. Marks inhales? Why?

 

Definition

1. The abdominal wall will move inwards

 

2. This occurs because the flaccid diaphragm is sucked up into the thorax by the negative pressure generated by the accessory muscles of inspiration.

Term

Mrs. White is a 70 year old woman who developed severe substernal chest pain radiating to her jaw and arm, associated with severe shortness of breath. An EKG revealed a large acute myocardial infarction. Chest radiographs confirmed the clinical suspicion of pulmonary edema. Arterial blood gases revealed a pH of 7.15, PCO2 of 60, and PO2 of 40. A sample of mixed venous blood drawn through a pulmonary artery (Swan-Ganz) catheter placed to aid management showed had an oxygen saturation of 50% (normal saturation is 75%).


21.          Why is Mrs. White acidotic?

Definition

1. There is a mixed metabolic and respiratory acidosis

 

2. (the decrease in pH is greater than that expected for the increase in PCO2.

 

Term

Mrs. White is a 70 year old woman who developed severe substernal chest pain radiating to her jaw and arm, associated with severe shortness of breath. An EKG revealed a large acute myocardial infarction. Chest radiographs confirmed the clinical suspicion of pulmonary edema. Arterial blood gases revealed a pH of 7.15, PCO2 of 60, and PO2 of 40. A sample of mixed venous blood drawn through a pulmonary artery (Swan-Ganz) catheter placed to aid management showed had an oxygen saturation of 50% (normal saturation is 75%).

 

 

22-23.    List two reasons why Mrs. White has a low mixed venous oxygen tension.

 

 

Definition

1. She has a low cardiac output and low oxygen saturation

 

2. Both of which decrease tissue oxygen delivery.

 

Term

Mrs. White is a 70 year old woman who developed severe substernal chest pain radiating to her jaw and arm, associated with severe shortness of breath. An EKG revealed a large acute myocardial infarction. Chest radiographs confirmed the clinical suspicion of pulmonary edema. Arterial blood gases revealed a pH of 7.15, PCO2 of 60, and PO2 of 40. A sample of mixed venous blood drawn through a pulmonary artery (Swan-Ganz) catheter placed to aid management showed had an oxygen saturation of 50% (normal saturation is 75%).


24-25.    Mrs. White has a normal hemoglobin level. Would transfusing her to higher levels increase her mixed venous oxygen? Why?

 

Definition

1. It would not

 

2. This is because the decreased cardiac output secondary to increased blood viscosity would negate the effects of the increase in hemoglobin on tissue oxygen delivery.

 

Supporting users have an ad free experience!